Modal Ontolojik Argüman ve İmkân Öncülü Argümanlarının Değerlendirilmesi – Utku Boyar

/
5 Okunma
Okunma süresi: 112 Dakika

Din felsefesinin Tanrı’nın varlığı tartışmaları çerçevesindeki zengin argüman spektrumunda diğerlerinden önemli bir noktada ayrılan bir argüman geleneği, ontolojik argümanlardır. En çok tartışılan argüman aileleri olarak teleolojik argümanlar evrende haklarındaki en iyi açıklamanın Tanrı’nın aktiviteleri olabileceği fenomenlerle, kozmolojik argümanlar bizzat evrenin varlığıyla veya kötülük argümanları dünyadaki kötülüğün varlığı/amaçsızlığıyla ilgilenirken bunların aksine ontolojik argümanlar bu tarz ampirik parçalara sahip değildir ve analitik, a priori öncüllerden yola çıkarak Tanrı’nın varlığını gösterme iddiasındadırlar.

Tarih boyunca birçok farklı şekilde formülize edilmiş olsa da burada özel olarak incelenecek olan ontolojik argüman, birçok çağdaş felsefecinin özellikle ilgi odağı olmuş modal ontolojik argümandır. Bir tanesi üzerine inşa edildiği modal mantık sistemi ve mükemmel varlık teizminin postulatlarıyla kanıtlanabilecek iki öncüle sahip argümanın iddiasını açık bir biçimde ortaya koyabilmek için bazı modal mantık kavramlarına yakından bakmalıyız. Böylece argümanın formülizasyonunu verecek, bahsi geçen öncülü bu modal mantık kavramları ve mükemmel varlık teizminin tanımlarıyla göstereceğiz. Bunların ardından anlaşılacağı gibi tartışmanın ilerlediği asıl nokta argümanın diğer öncülü olacak. Tüm bunları yapıp tartışma zeminini temizledikten sonra ise yazının asıl amacı bu öncül ve modal ontolojik argümanın “tersi” lehine argümanları tartışmak olacak.

1. Modal Mantık ve Ontolojik Argüman

1. Modal Kavramlar

Modal mantık, şeylerin nasıl olabileceği ile ilgilenir. Şu önermeleri örnek olarak inceleyebiliriz:

  • i. Şu an bu yazıyı okuyor olsanız bile bunun yerine yürüyüş yapıyor olabilirdiniz.
  • ii. İki kere ikinin dört ettiği yalnızca doğru değildir, bu zorunlu olarak doğrudur.
  • iii. Aynı anda hem beyaz hem de beyaz olmayan bir arabaya sahip olmamam yalnızca doğru değildir, böyle bir durum imkansızdır.

Tüm bunlar, şeylerin nasıl olup olamayacağı veya olmak zorunda olduklarıyla ilgili modal ifadelerdir. Bu noktada birkaç tanım yapabiliriz. Bir önerme doğru olabilirse o önermenin doğru olması mümkündür, aynı şekilde, bir önerme yanlış olabilirse o önermenin yanlış olması mümkündür. Bir önerme doğru olmak zorundaysa o önerme zorunlu olarak doğrudur, yine aynı şekilde bir önerme yanlış olmak zorundaysa o önerme zorunlu olarak yanlıştır (veya o önermenin doğru olması imkansızdır). Bu tanımlara dayanarak bir başka kavramı tanımlamak gerekirse, bir önerme gerçekte doğruysa ancak doğru olmak zorunda değilse –bir diğer deyişle zorunlu olarak doğru değilse- olumsal olarak (contingently) doğrudur.

Tanımlar ışığında yukarıdaki örneklere tekrar bakarsak söyleyebiliriz ki sizin yürüyüş yapıyor olmanız mümkündür, şu an bu yazıyı okuyor olmanız ise olumsal bir doğrudur. İki kere iki zorunlu olarak dörde eşittir ve bir nesnenin hem beyaz hem de beyaz olmaması mümkün değildir/imkansızdır.

1. 2. İmkan Türleri

Mümkün olma, zorunluluk ve olumsallığı tanımlamış olsak da kavramlar kullanıldıkları bağlamlara göre anlaşılabilmektedir. Bu bakımdan hangi imkan türü ile ilgilendiğimizi de açık bir biçimde ortaya koymalıyız. Her şeyden önce ifade etmeliyiz ki bahsettiğimiz ve modal ontolojik argümanın da nesnesi olan imkan türü epistemolojik imkan değildir. Bir önermenin doğru olmasının mümkün olduğunu söylerken sahip olduğumuz bilgiler ışığında o önermenin doğru olabileceğini kastetmiyoruz. Neyi kastettiğimizi açıklamak için yine birkaç örneği inceleyelim:

  • iv. Işık hızını aşabilecek bir uçak mümkündür.
  • v. Zorunlu olarak, Sokrates ya bir insandır ya da değildir.
  • vi. Zorunlu olarak, var olmaya başlayan her şeyin bir sebebi vardır.

Işık hızı aşılamaz olduğuna göre ilk önermenin yanlış olduğunu söylemeliyiz. Ancak kastedilen bunun fiziksel imkanı değil de örneğin mantıksal imkanıysa önerme doğru görünmektedir zira ışık hızının aşılması fizik yasalarınca imkansızken basitçe hiçbir mantık yasasını ihlal etmediğinden mantıksal olarak mümkündür. Diğer taraftan ikinci önerme mantıksal olarak da doğrudur zira Sokrates’in hem bir insan olup hem de insan olmaması çelişmezlik yasasını ihlal etmektedir. Fiziksel imkana geri dönersek, tartıştığı durumun fizik yasalarına aykırı olması dolayısıyla fiziksel imkan kastedildiğinde yanlış olan ilk önermenin yanında ikinci önerme fiziksel imkan kastedilse de doğrudur zira bir önermenin mantıksal olarak imkansız olması fiziksel olarak da imkansız olduğu anlamına gelir.

Fiziksel imkan fizik yasaları, mantıksal imkan mantıkla ilişkili iken metafiziksel imkan var olabilecek şeylerin doğalarının izin verip vermemesiyle ilgilidir. Bu anlamda bakıldığında üçüncü önerme bu bağlamda anlaşılmalıdır. Eğer hakikaten sebebe sahip olmaksızın varlığa gelmek mümkün değilse önerme doğrudur. Fiziksel olarak mümkün olmayan durumlar metafiziksel olarak mümkün olabilir. Örneğin doğa yasaları farklı olsaydı ışık hızını aşmak mümkün olabilirdi, bu anlamda ilk önerme –doğa yasaları zorunlu değilse- metafiziksel anlamda doğrudur. Öte yandan “2+2=4”, “Evli bir bekar imkansızdır.” gibi önermeler mantık yasalarından dolayı değil ancak şeylerin doğaları gereği doğrudur. Bu bakımdan aşağıdaki şema, modal mantıkta imkan türlerine ilişkin standart modeli yansıtmaktadır:

1

Teistlerce Tanrı’nın zorunlu varlık olduğu ifade edildiğinde metafiziksel zorunluluktan bahsedilmektedir. Modal ontolojik argümanın ele aldığı ve bizim de yazı boyunca tartışacağımız imkan türü metafiziksel imkan olacak.

1. 3. Mümkün Dünyalar Semantiği

Modal kavramlardan, önermelerin modal statülerinden bahsetmenin analitik felsefeciler arasında kullanılan bir yolu mümkün dünyalar semantiğidir. Bir önermenin doğru olmasının mümkün olması, o önermenin bazı mümkün dünyalarda doğru olduğu ile ifade edilir. Benzer şekilde bir önermenin zorunlu olarak doğru olmasının mümkün dünyalar dilinden ifadesi o önermenin tüm mümkün dünyalarda doğru olduğu şeklindedir. Anlaşılacağı gibi bir önermenin olumsal olması ise bazı mümkün dünyalarda doğru ve bazılarında yanlış olduğu anlamında gelir. Burada mümkün dünyalar ile olası evrenleri değil gerçekliğin alabileceği bütün bir tabloyu kastettiğimizi vurgulamak önemlidir. Mümkün bir dünyanın ne anlama geldiğini tam anlamıyla açıklamak için iki ön tanım yapabiliriz. Bir A durumunun geçerli olup B durumunun olmaması mümkün değilse A’nın B’yi içerdiğini (include) söyleriz. A durumu ile B durumunun aynı anda geçerli olması mümkün değilse de A ve B birbirini dışlar (preclude). Örneğin Jim Whittaker’ın Everest’e tırmanan ilk kişi olması Everest’e tırmanılmış olmasını içerirken Luther Jerstad’ın Everest’e ilk tırmanan kişi olmasını dışlar.2 Tanımlar ışığında ise mümkün bir dünya tüm durumlar için onları ya içeren ya da dışlayan toplam durumdur.

Mümkün dünyalarla ilgili olan bir diğer kavram ise kitaplardır. Her W mümkün dünyası için, W’de doğru olan tüm önermelerden –bir diğer ifadeyle eğer W fiili olsaydı doğru olacak olan ve W fiili iken yanlış olması mümkün olmayan tüm önermelerden- oluşan küme için W’ye karşılık gelen kitap ifadesi kullanılır. Mümkün dünyalara benzer olarak kitaplar da her önerme için ya o önermeyi ya da değillemesini içerir. Her mümkün dünyanın kendisine karşılık gelen bir tane kitabı vardır ve tersi de geçerlidir.

Mümkün dünyalar arasından bir tanesi ise fiili dünyadır (actual world). Yalnızca bir mümkün dünya fiilidir ve bir önermenin gerçekte doğru olduğunu söylemek o önermenin fiili dünyada doğru olduğu anlamına gelir.

1. 4. S5 Aksiyomu ve Mükemmel Varlık Teizmi

Modal ontolojik argümanın kullandığı modal mantık sisteminin önemli bir parçası, argümanın basamaklarından birini de oluşturan S5 aksiyomudur. Aksiyom mümkün dünyaların birbiri için erişilebilir (accessible) olduğu kabulünü içerir. Buna göre önermelerin modal statüleri –önermelerin mümkün/zorunlu oluşları- mümkün dünyalar arasında farklılık göstermez. <Mümkündür ki P> veya <Zorunlu olarak P> gibi önermeler tüm mümkün dünyalarda aynı doğruluk değerine sahiptir. Bir P önermesinin doğru olmasının imkan dahilinde oluşunun P’nin doğru olduğu mümkün bir dünyanın bulunduğu anlamına geldiğini görmüştük. Benzer olarak P zorunlu olarak doğruysa tüm mümkün dünyalarda doğru idi. Öyleyse söyleyebiliriz ki P’nin zorunlu olarak doğru olması mümkünse P zorunlu olarak doğrudur –P mümkün bir dünyada zorunlu olarak doğruysa, dünyalar erişilebilir olduğu için, P tüm mümkün dünyalarda doğru olacaktır. S5 aksiyomunun ifadesi basitçe budur.

Plantinga, mükemmel varlık teizmine dayanarak Tanrı’nın var olması durumunda zorunlu olarak var olacağını ifade etmiştir. O, maksimum derecede üstünlüğün (maximal excellence) mümkün bir dünyada örneklenmesini o mümkün dünyada kadir-i mutlak, alim-i mutlak ve ahlaken kusursuz bir varlığın bulunması; maksimum derecede mükemmelliği (maximal greatness) ise maksimum derecede üstünlüğün tüm mümkün dünyalarda örneklenmesi şeklinde tanımlamıştır. Mükemmel varlık teizminin, Tanrı’nın kendisinden metafiziksel anlamda daha yücesi mümkün olmayan varlık olduğu şeklindeki doktrini göz önüne alındığında Tanrı’nın maksimum derecede mükemmel varlık şeklinde tanımlanabileceğini söyleyebiliriz. Ancak öyleyse Tanrı ya tüm mümkün dünyalarda var olmalı ya da hiçbirinde var olmamalıdır –Tanrı’nın varlığı ya zorunlu ya da imkansızdır. Dolayısıyla bu tanımdan ve S5 aksiyomundan çıkarabiliriz ki Tanrı’nın var olması mümkünse (mümkün bir dünyada Tanrı varsa), zorunlu olarak Tanrı vardır (tüm mümkün dünyalarda vardır).

1. 5. Modal Ontolojik Argüman 

Geldiğimiz nokta itibariyle modal ontolojik argümanı mümkün dünyalar semantiğini kullanarak şu şekilde ifade edebiliriz:

  • (1) Mümkün bir dünyada Tanrı vardır.
  • (2) Mümkün bir dünyada Tanrı varsa tüm mümkün dünyalarda Tanrı vardır.
  • (3) Öyleyse Tanrı vardır.

Argüman başlangıçta da belirttiğimiz gibi iki öncüle sahip: (1) ve (2). Ek olarak söz ettiğimiz üzere öncüllerden biri olan (2) S5 aksiyomu ve mükemmel varlık teizmine dayanan güçlü bir argümanın da işaret ettiği üzere doğru görünüyor. Her ne kadar S5 aksiyomu ve oluşturduğu modal mantık sistemine veya mükemmel varlık teizminden yola çıkan tanıma itiraz edilebilecek olsa da bu öncül neredeyse bir konsensüs haline geldiği ve asıl münazara (1) etrafında yapıldığı için biz de odağımızı bu öncülde toplayacağız.

O halde argümanın imkan öncülü (possibility premise) olarak da anılan ilk öncülü için ne söyleyebiliriz? İlk bakışta makul görünse de modal ontolojik argümana verilebilecek doğal bir tepki onun tersini kurmak olacaktır.

  • (4) Mümkün bir dünyada Tanrı yoktur. (Tanrı’nın var olmaması mümkündür.)
  • (2) Mümkün bir dünyada Tanrı varsa tüm mümkün dünyalarda Tanrı vardır.
  • (~3) Öyleyse Tanrı yoktur.

Son tahlilde yalnızca ilk öncülleri olan (1) ve (4) ile farklılaşan ve zıt sonuçlara ulaşan bu iki argümanın hangisinin daha makul olduğuna karar vermek için yine makul oluşları bakımından aralarında –ek değerlendirme yapılmaksızın- bir simetri var gibi görünen (1) ve (4)’ten hangisinin daha makul olduğuna bakmalıyız. Her ne kadar Plantinga argümanı ortaya koyduğunda (1) lehine pozitif bir argüman verme yoluna gitmemiş olsa da başkaları tarafından bu oldukça farklı ve yaratıcı yöntemlerle yapılmıştır. Gelecek bölümde biz de bu argümanları değerlendiriyor olacağız. Sonraki bölümü ise (4) lehine iki argümanı tartışmaya ayıracağız.

2. İmkan Öncülü Lehine Argümanlar

2.1. Tasavvur Edilebilirlik Argümanı

Başlangıç için akla muhtemelen ilk sırada gelecek akıl yürütmeyi inceleyebiliriz. Önermelerin mümkün olup olmadıklarını incelemede kullanılan, kullanılmasının meşru olup olmadığı da ayrıca tartışılan bir argüman türüne göre doğru olduğunu tasavvur edebildiğimiz önermelerin doğru olması mümkündür. Tanrı’nın var olduğunu tasavvur edebiliyoruz. O halde Tanrı’nın var olması mümkün olmalıdır.

Argümanın her iki öncülünün de ciddi sorunları bulunuyor. Bu tip bir argümanı savunma yoluna gidecek birinin doğrudan Tanrı’nın mümkün olduğunu göstermek yerine bunu tasavvur edilebilirliğe atıf aracılığıyla yapmayı seçmesinin, tasavvur edilebilirlik ile imkan arasında bir boşluk olduğunu ima ettiği gözlemi ile ilk sorunu incelemeye başlayabiliriz.3 Bu boşluğun büyüklüğü ise öznenin tasavvur edilebilirliği incelenen varlığı tasavvur etmedeki kabiliyetiyle ilişkilidir. Bu nokta bizi Chalmers’ın prima facie ve ideal tasavvur edilebilirlik ayrımına götürüyor.4 Buna göre bir kavram özne için ilk bakışta tasavvur edilebilir görünüyorsa kavram özne için prima facie tasavvur edilebilirdir. Diğer taraftan kavram özneye ideal rasyonel kavrayışla beraber tasavvur edilebilir görünüyorsa onun için ideal olarak tasavvur edilebilirdir. Chalmers’ın örneğine göre doğru olup olmadığı henüz bilinmeyen ancak ileride kanıtlanacak olan matematiksel bir ifadenin yanlışlığı prima facie tasavvur edilebilir olacakken ideal olarak tasavvur edilebilir olmayacaktır. Bu noktada imkanı gerektiren tasavvur edilebilirlik türü prima facieden ziyade ideal tasavvur edilebilirlik gibi görünüyor zira kavramı hakkıyla değerlendirmeye kadir olmayan bir özne onun içindeki -eğer varsa- tutarsızlığı fark etmeye de aynı şekilde kadir olmayabilir. Öyleyse argümanın savunucuları Tanrı’nın varlığını ideal olarak tasavvur edebildiğimizi göstermelidirler ancak bu kolay değildir.

Tanrı hakkında ideal tasavvur edilebilirliği göstermenin kolay olmamasının bir nedeni aynı argümanın ters ontolojik argüman için de kurulabilecek olmasıdır. Aynı şekilde Tanrı’nın var olmamasını da tasavvur edebiliyoruz. Öyleyse Tanrı’nın var olmaması da mı mümkün olmalıdır? Bu noktada argümanın savunucuları Tanrı’nın var olmamasını aslında tasavvur edemediğimizi zira onun zorunlu varlık olduğunu söyleyebilirler ancak zorluk tam olarak buradadır. Ek bir temellendirmeye dayanmayan bu tarz bir hamle argümanı döngüsel hale getirir.

Tasavvur edilebilirliğin imkanı gerektirdiğine dair öncülün problemi ise hangi tür imkanı gerektirdiğinin kuşkulu olmasıdır. Öyle görünüyor ki tasavvur edilebilirlik metafiziksel imkandan ziyade içsel tutarlılığa işaret ediyor olabilir. Tasavvur edilebilirlik argümanını zorlanmadan, hatta Tanrı için olandan çok daha kolay biçimde, gereksiz kötülük gibi teizmle bağdaşmayan bir kavram için de kurabiliriz. Tartışma açısından önemli olan nokta şu ki bunu Tanrı var olsa dahi yapabileceğimiz iddia edilebilir –bunu normal şartlarda yapabileceğimiz ancak Tanrı’nın varlığı durumunda yapamayacağımızı söylemek sorunlu görünmektedir. Ancak Tanrı’nın var olması durumunda gereksiz kötülük metafiziksel açıdan mümkün olmayacaktır. O halde onu tasavvur ederek ne açıdan mümkün olduğunu göstermiş olabiliriz? Muhtemelen gereksiz kötülüğü (en azından argümanı savunacak birinin ifade edeceği şekilde) tasavvur ederek onun içsel bir çelişki içermediği, dolayısıyla da mantıksal olarak mümkün olduğu sonucuna ulaşabiliriz. Öyleyse söyleyebiliriz ki tasavvur edilebilirlik ile varılabilecek sonuç muhtemelen ihtiyacımız olan metafiziksel imkan değildir, dolayısıyla da argümanın bu öncülü de zayıftır.

2.2. Tecrübe Edilebilirlik Argümanı

Alexander Pruss ve Brian Leftow gibi isimlerin ortaya koyduğu oldukça ilginç ve orijinal bir argüman, teizm lehine öne sürülegelmiş bir başka ilginç argüman geleneği olan dini tecrübeden destek alır.5 Yüzyıllar boyunca birçok mistik, aziz ve derviş Tanrı’yı içeren deneyimler yaşadıklarını bildirmiştir. Her ne kadar bu iddialardan yola çıkan dini  tecrübe argümanlarına bu deneyimlerin halüsinasyondan ileri geldiği ifade edilerek cevap veriliyor olsa da Pruss’un sekizinci yüzyıl Hint filozofu Śaṃkara’nın ismiyle andığı ilke ışığında dini tecrübeler imkan öncülü için iyi bir kanıt sağlayabilir: Eğer x imkansızsa deneyimlerde -mistik olanlar da dahil- yer alamaz. Örneğin kimse halüsinasyon yolu ile dahi olsa yuvarlak kare veya evli bekar deneyimleyemez.  Yani x deneyimlerde yer alıyorsa mümkündür. Öyleyse Tanrı birçok mistik deneyimde yer aldığına göre –söz konusu deneyimler hakiki olsun veya olmasın- mümkün olmalıdır.

Problematik olan temel noktalardan biri mistiklerin tecrübelerinin hakikaten maksimum derecede mükemmel (1.4. bölümdeki tanım) bir varlığa dair olup olmadığıdır. Argümanın çalışması için bahis konusu tecrübelerin maksimum derecede mükemmel varlık kavramıyla birebir örtüşmesi gerekir. Zira örneğin her ikisinin de küçük bir bölümü net bir şekilde gözlemlenemeyen ancak kalan bölümleri tamamen aynı olan iki nesneye dair bir deneyim, tüm özellikleri aynı olan farklı iki nesnenin var olamayacağını söyleyen ayırt edilemezlerin özdeşliği ilkesinin (principle of identity of indiscernibles) hatalı oluşuna -gözlemlenemeyen bu bölümler dolayısıyla- yeterli bir örnek oluşturmazdı. Paralel olarak dini tecrübelerin kadir-i mutlak, alim-i mutlak, ahlaken kusursuz zorunlu bir varlığı içeriyor olması gerekir ancak bu denli spesifik ve sonsuz derecedeki niteliklerin kusurlu algılara sahip sonlu varlıklarca ayırt edilebilmesi güç görünmektedir. Mistiklerin kadir-i mutlak bir varlıkla çok güçlü, alim-i mutlak bir varlıkla oldukça bilgili bir varlığı ayırt etme gücünde olduklarını iddia etmek oldukça büyük bir kanıt yükünü sırtlanmak olacaktır. Öyle ki maksimum derecede mükemmel bir varlık tutarsız ancak oldukça mükemmel bir varlık mümkün olabilir ve hala mistikler bu tarz varlıkların deneyimine sahip olup bunları maksimum derecede mükemmellikle karıştırıyor olabilirler. Deneyimin maksimum derecede mükemmel varlık kavramıyla uyumlu olması yeterli değildir, birebir örtüşme gerekmektedir.

Öte yandan sorunun bundan daha ciddi olduğunu da söyleyebiliriz. Maksimum derecede değil ancak oldukça mükemmel varlık deneyimleri, gösterilmek istenenle uyumlu olmayabilir de. Örneğin Plantinga neredeyse maksimum derecede mükemmel (nearly maximally perfect), kendi tanımıyla, tüm mümkün dünyalarda bulunmayan ancak tüm mümkün dünyalardaki tüm varlıklardan daha üstün bir varlığın maksimum derecede mükemmel bir varlıkla bağdaşmayacağını  düşünmüştür. Öyleyse mistiklerin tecrübelerinin bu tarz olumsal bir varlığı içeriyor olması durumunda argümanın uygun şekilde desteklenmeyecek olması bir yana, argüman ters modal ontolojik argüman lehine dahi dönebilir. Benzer ve belki de daha ciddi bir sorun Saikat Guha’nın dikkat çektiği üzere Doğu mistiklerinin deneyimleri zorunlu bir varlık olarak kişisel olmayan (non-personal) nihai bir gerçekliği (ultimate reality) içermektedir. Argümana göre bu da Doğu mistiklerinin tecrübe ettiği nihai gerçekliğin hakikati yansıttığı anlamına gelmelidir ancak bu teistik argümanın sonucuyla tutarsızdır. Dolayısıyla ikisi birden doğru olamaz. Bu noktada Pruss, hem Doğu hem de Batı mistiklerinin tecrübelerinde ortak olan noktaya bakmamız gerektiğini ifade eder. Ona göre mistiklerin tecrübe ettiği maksimum mükemmelliktir ve bazı mistikler maksimum derecede mükemmelliğin gerektirdiğini düşündükleri nitelikler dolayısıyla tecrübelerini yanlış yorumlamışlardır. Maksimum derecede mükemmelliğin hangi nitelikleri içerdiğini incelediğimizde ahlaken kusursuz olma gibi ancak kişisel varlıklarca sahip olunabilecek özelliklerle karşılaşırız. Dolayısıyla da Doğu mistiklerinin deneyimleri hakiki ve dolayısıyla aslında teistik argümanı destekler niteliktedir. Diğer taraftan tecrübe tariflerindeki farklar mistiklerin yanlış maksimum mükemmellik anlayışlarından ileri gelmektedir.

Pruss’un önerisi kabul edilecek olursa en başta değinilen itiraz güç kazanacaktır. Eğer gerçekten de Doğu mistikleri maksimum derecede mükemmel varlık deneyimlerine sahipse ve bunu kadir-i mutlak, alim-i mutlak, ahlaken kusursuz kişisel bir varlık olarak değil de kişisel olmayan nihai bir gerçeklik olarak yorumluyorsa, farklı medeniyetlerdeki mistikler bu kadar kökten farklılıklara sahip tecrübe yorumlamaları yapıyorken, mistiklerin (Doğu veya Batı) nihayetinde maksimum derecede mükemmel bir varlık deneyimine sahip olduğunu düşünmek için bir neden var mıdır? Öyle görünüyor ki aynı varlığın tecrübesine sahip oldukları halde kişisellik gibi en temel niteliklerden birinde dahi ihtilafa düşen mistiklerin tecrübelerinde mutlak derecede güçlü, bilgili, iyi olduğunu beyan ettikleri varlığın mutlak değil ancak oldukça güçlü, bilgili, iyi olabileceğini düşünmek makuldür. Tüm bunlardan dolayı tecrübelerin Tanrı kavramıyla birebir örtüşüyor olması son derece ihtimal dışı olarak karşımıza çıkmaktadır.

Diğer taraftan tasavvur edilebilirlik argümanının sahip olduğu bir sorunla burada da karşı karşıyayız. Argümanın Tanrı’nın varlığının metafiziksel olarak mümkün olduğunu gösterdiği yeterince açık değildir. Örneğin fizik yasalarının zorunlu yasalar olduğu bir durumda dahi halüsinasyon tarzı bir yolla fizik yasalarını ihlal eden olaylara tanık olabiliriz. Bu durumda tecrübemizin bize vereceği sonuç tanık olduğumuz olayın içsel olarak tutarsız olmadığı olacaktır, gerçekleşebileceği değil. Bir diğer örnek yeniden Tanrı ve onunla tutarsız bir durum için verilebilir. Maksimum mükemmellikteki bir varlığın var olması durumunda bu varlık tecrübe edilebiliyorken onun bir yerde olumsal hali olan neredeyse maksimum derecede mükemmel bir varlığın yine halüsinasyon benzeri bir süreçle tecrübe edilemeyeceğini ifade etmek makul olmayacaktır. Ancak neredeyse mükemmel bu ikinci varlık, yukarıda bahsi geçtiği üzere ilkiyle tutarlı olmadığına göre aslında var olamaz. Dolayısıyla da onun gerçek olmayan bir tecrübesine sahip olmamızdan yola çıkan argümanın verebileceği sonuç bu varlığın yalnızca içsel olarak tutarlı veya en fazla mantıksal olarak mümkün olduğu olacaktır, yeniden, var olabileceği değil.

Pruss’un Śaṃkara ilkesi lehine verdiği bir argüman, son itirazımızı desteklemesi açısından incelenebilir. Bir deneyimin bütünlük içerisinde vuku bulabilmesi için tutarlı olması, çelişen parçalara sahip olmaması gerekir. Bu fikir hattında Pruss ile uyum içinde görünüyoruz. Bizce de bir kavrama ilişkin deneyimin varlığı, o kavramın tutarlı olduğu anlamına gelecektir. Ancak bir kavramın, önermenin yahut varlığın tutarlı olmasının onun var/doğru olabileceğini gösterdiğini ifade etme anlamında tutarlılığın (coherence) imkanı gerektirdiğini reddediyoruz. Bir kavramın içsel olarak tutarlı olması onu mantıksal olarak mümkün kılabilir ancak onun metafiziksel imkanını, örneklerde gösterilmeye çalışıldığı üzere, garanti etmez.

2.3. Deontik Mükemmellik Argümanı

Carl Kordig, Tanrı’nın mümkün bir dünyada var olduğuna yönelik oldukça ilginç ve orijinal bir argüman geliştirmiştir.6 Maksimum mükemmelliğinin bir gereği olarak, Kordig’e göre Tanrı özelde deontik olarak mükemmeldir. Deontik olarak mükemmel (deontically perfect) bir varlık ise var olmalıdır (ought to exist). Var olması gereken bir varlık ise var olabilir (can exist). Öyleyse Tanrı var olabilir.

Halihazırda karmaşık görünürken Türkçe ifadesi işleri daha da karıştırabilecek argüman bu şekildedir. Argüman deontik mükemmellik olarak anılan bir nitelik tanımlar ve bu niteliğin, Kant’ın gerekliliğin/sorumluluğun imkanı gerektirdiğine dair yasasına (ought implies can) atıfla deontik mükemmelliğe sahip Tanrı’nın var olmasının mümkün olduğu sonucuna ulaşır. Bu anlamda argümanın bir modal mantık türü olan deontik mantığın araçlarını kullandığı söylenebilir. Kordig her ne kadar deontik mükemmelliği net biçimde tanımlamamış olsa da “ateistlerin dahi Tanrı’nın var olması gerektiğini teslim etmeleri gerektiğine” dair ifadeleri doğrultusunda söz  konusu nitelik hakkında bir dereceye kadar sezgisel bir fikir sahibi olabiliriz. Bu noktaya aşağıda yeniden döneceğiz.

Argümana ilk itiraz, maksimum derecede mükemmelliğin deontik mükemmelliği gerektireceğini kabul etsek dahi maksimum derecede mükemmelliğin bunun dışında birçok niteliği de içerdiğine dikkat çekerek ortaya konabilir. Önce Tanrı’dan başka bir örnek düşünelim. Deontik mükemmellik kişilerin sahip olabileceği bir nitelik gibi görünüyor; öyleyse deontik olarak mükemmel, sihirli bir şifacı tahayyül edebiliriz. Bu şifacı doğa yasalarını aşkın yöntemlerle hastaları tedavi ediyor olsun. Şifacının deontik mükemmellik niteliğine sahip olması onun mümkün bir varlık olduğu anlamına gelir mi? Zorunlu olarak değil. Belki de deontik mükemmellikten bağımsız olarak sihirli şifacı kavramı tutarsızdır. Örneğin belki de doğa yasaları olumsal değil zorunludur ve bu yüzden sihirli şifacı mümkün değildir ve sihirli şifacı kavramında içsel bir tutarsızlık vardır. Benzeri Tanrı için de geçerli olabilir. Eğer maksimum mükemmellik kavramı bir iç tutarsızlık içeriyorsa deontik olarak mükemmel olduğuna bakmaksızın Tanrı’nın var olmasının mümkün olmadığını söyleriz. Bu anlamda argüman bir bakıma tamamlanmış değildir.

Argümanın muhtemelen daha ciddi sorunu kavramların kullanımı ile ilgilidir. Denontik mükemmellik ile ne kastettiğini açık bir biçimde ortaya koymamış olsa da Kordig ateistlerin dahi Tanrı’nın var olması gerektiğini teslim etmeleri gerektiğinden bahsetmektedir. Tanrı’nın var olmasının gerekliliği ne tür bir gereklilik olabilir? Kant yasasının ilgilendiği gereklilik, deontik mantığın bir operatörüdür. Örneğin gerekliliği ifade eden <Tom röntgencilik yapmamalıdır.> önermesi, Kant yasasına göre <Tom’un röntgencilik yapmaması mümkündür.>’ü gerektirir. Bir başka örnek daha vermek gerekirse <Deniz borcunu ödemelidir.> yasaya göre <Deniz’in borcunu ödemesi mümkündür.> önermesini gerektirir. Bu açıdan bakıldığında <Tanrı var olmalıdır.> önermesi farklı bir konumda gibi görünüyor. Yasanın geçerli olabileceği örneklerde bir anlamda sorumluluk tarzı bir gereklilikten bahsediliyorken Tanrı’nın var olması gerektiğine dair önermede bu -ki Kordig’in ateistlerin bu konudaki tavrı  hakkındaki beklentisi de bu yöndeki fikrimizi destekler- daha ziyade istekle alakalıdır. Bir diğer ifadeyle Tanrı var olmalıdır derken Tanrı’nın var olması istenilir bir durumdur demiş oluyoruz. Ancak bu Kant’ın yasasındaki, daha çok eylemlere ve bunlarla ilgili sorumluluklara dair gereklilikten farklıdır. Bir şeyin var olmakla sorumlu olmasının zaten başlı başına absürt olduğu da iddia edilebilir.

2.4. Reformist Epistemoloji

Teist inancı yaratılanlar üzerine akıl yürütmelere dayanan temellendirmelerle gerekçelendirme geleneği olan doğal teolojiden farklı olarak, Hristiyanlık içerisindeki reformist hareketin öncülerinden olan Calvin’in oluşturduğu Kalvinizm ve çağdaş felsefede Alston, Plantinga ve Wolterstorff gibi felsefeciler tarafından bu yaklaşımdan yararlanılarak ortaya konmuş bir başka gelenek olan reformist epistemolojiye göre Tanrı’ya inanç delile gerek duymaksızın rasyoneldir. Kalvinistler tüm insanların yaratılıştan gelen bir ilahi his (sense of deity veya sensus divinitatis) sahibi olduklarını söyler. İmanın nasıl rasyonel olduğu konusunda birbirinden farklı anlayışlara sahip gelenekler olsalar da Daniel Johnson’a göre Kalvinist anlayışın (veya çağdaş savunucularının adlandırdığı haliyle reformist epistemolojinin), doğal teolojinin bir parçası olabileceği durumlar söz konusu olabilir. Johnson reformist epistemolojinin enstrümanlarından faydalanarak modal ontolojik argüman ve Leibnizci kozmolojik argümanın kritik öncüllerini tartışır.7 Büyük ölçüde benzer bir çizgiden ilerleyen tartışmalar arasından burada konumuz gereği ontolojik argümanla alakalı olan ile ilgileneceğiz.

Diyelim ki Bernie Tanrı’nın var olduğuna inanmaktadır. Bunun doğal bir sonucu olarak o aynı zamanda Tanrı’nın var olmasının mümkün olduğuna da inanacaktır. Üstelik Bernie’nin Tanrı’nın varlığına inancı ilahi hisse dayanan rasyonel bir inançtır. Ancak Bernie ne yazık ki –Calvin’in inançsızlığı bağladığı üzere- günahkarlık gibi rasyonel olmayan (non-rational) nedenlerle ilahi hissini baskılamıştır ve Bernie artık Tanrı’ya inanmamaktadır. Ancak, der Johnson, Bernie Tanrı’nın varlığının mümkün olduğuna inanmayı hala sürdürebilir ve onun bu inancı hala rasyonel, gerekçelendirilmiş de olabilir. Tüm bunların ardından Bernie modal ontolojik argümanla karşılaştığında imkan öncülü –Tanrı’nın varlığının mümkün olduğunu söyleyen öncül- onun için gerekçelendirilmiş olacaktır. Böylece Bernie Tanrı’ya yeniden modal ontolojik argüman ile birlikte rasyonel olarak inanabilir.

Johnson’ın da belirttiği üzere argümanın dayandığı üç nokta bulunuyor. Birincisi, Tanrı’ya inanmayı bırakan kişilerden en azından bazılarının bu inanç kaybı rasyonel temellere dayanmamalıdır –ve esasen, argüman bu kişiler için geçerli olacaktır. Calvin Tanrı’ya inanmayan herkesin bu durumda olduğunu düşünse de Johnson daha mütevazı bir tutum benimser ve söylenenin en azından bazı kişiler için doğru olduğunu kabul eder. İkincisi, bir inancı rasyonel olarak sürdürmenin o inancın temelleri rasyonel olmayan (unutma gibi) bir süreçle yitirilse dahi mümkün olması gerekir. Örneğin Pisagor teoreminin ispatını unutmuş olsanız dahi teoreme inanmayı sürdürmek sizin için rasyoneldir. Söz konusu tartışmaya daha çok yakınsayan bir örnek: otel oda numaranız 314 ise ve bunu kafanızda pi sayısının ilk üç rakamı olarak kodladıysanız, oda numaranızı ve dolayısıyla kafanızdaki bu kodun temelini unuttuğunuzda yine de oda numaranızın pi’nin ilk üç rakamı olduğunu hatırlayıp oda numaranızın 314 olduğunu çıkarabilirsiniz. Burada önemli olan, diğerine de temel olan ilk inancın rasyonel olmayan nedenlerle yitirilmesidir. Argümanın işleyebilmesi için  Tanrı’ya inanmayı bırakma nedenlerinin de günahkarlık veya dünyevi hazlara düşkünlük gibi “rasyonel olmayan” temeller olmasının gerekmesi bu yüzdendir. Üçüncüsü, ilahi hissin Tanrı’nın zorunlu varlık oluşunu da kapsaması gerekir.

Argüman hakkında söylenebilecek ilk şey imkan öncülünün doğruluğu için bir argüman olmasından ziyade gerekçelendirilmiş bir inanç olabileceğine yönelik bir argüman olmasıdır. Bir önermeye inanmak gerekçelendirilmiş, rasyonel iken önerme yine de yanlış olabilir. Argümanın bu özelliği aynı zamanda onu şimdiye kadar tartıştığımız ve bundan sonra tartışacağımız argümanlardan da ayırır. İkinci bir nokta, Johnson’ın da belirttiği üzere argümanın yalnızca bazı kişiler için etkili olabileceğidir. Bu bazı kişilerin inançlarını rasyonel olmayan bir süreçle kaybetmiş olmaları özellikleri dolayısıyla az sayıda oldukları Calvin’in aksine iddia edilebilir. Bir argümanın değeri ikna edebileceği kişilerin sayısı ve bu kişilerin rasyonelliği ile de ilişkili ise argüman, hem nispeten az kişiye hitap edeceğinden hem de bu kişilerin bir düzeye kadar irrasyonel oluşlarından kaynaklı olarak kusurludur denebilir. Üçüncü nokta, sürecin tersten işleyebileceğini de söyleyebilmemiz mümkündür. Bir kişi Tanrı’nın var olmadığına inanıyorken rasyonel olmayan nedenlerle (ölümden korkma veya gerçekten rasyonel olup olmadıklarını konudan uzaklaşmamak adına tartışamayacağımız başka örneklerdeki gibi) bu inancını terk edip Tanrı’ya inanmaya başlayabilir. Böyle bir kişi için ontolojik argüman ters sonucu verecektir. Üçüncü noktanın önemli bir şartı reformist epistemolojinin Tanrı’nın varlığına inanca atfettiği uygun temel (properly basic) olmanın Tanrı’nın var olmamasına inançta da geçerli olmasının gerekebilmesidir. En azından yalnızca teizmin uygun temel inanç olabileceği gösterilmediği müddetçe bu senaryoyu da kabul edebiliriz.

Argümanla ilgili dördüncü ve en önemli sorun, modal ontolojik argümanın hakikaten kullanılıyor olup olmadığının şüpheli olmasıdır. Bernie üzerinden tasvir edilen senaryoyu önermesel olarak yeniden inceleyelim.

  1. Tanrı vardır.
  2. Mümkündür ki Tanrı vardır.
  3. B ise A.

Bernie A’ya inanır. Bunun doğal sonucu olarak B’ye de. Ancak ne yazık ki Bernie A’ya inancını rasyonel olmayarak bastırır/yitirir. Ama bu yine de onun –muhtemelen bilinçli olmayarak- B’ye inanmayı sürdürmesine izin verir. Daha sonra Bernie C’yi öğrenir. Böylece yeniden A’ya inanmaya başlar.

Sürece yakından bakalım. C’yi öğrendiğinde Bernie halihazırda ~A ve B’ye inanmaktadır. C’ye inanmaya başlaması ona aynı anda ~A ve B’ye inanmaya daha fazla devam edemeyeceğini düşünmek için neden verir. Öyleyse o ne yapacaktır? Senaryoya göre ~A’ya inanmayı bırakmalıdır. Peki neden B yerine ~A inanmayı bırakmalıdır? Bernie yerinde olmayan (ve Tanrı’nın varlığının yine de mümkün olduğunu düşünen) bir ateist, diyelim ki Joe, ontolojik argümanla karşılaştığında imkan öncülünün –burada B’nin- yanlış olduğunu söyleyecektir. Joe için söz konusu olan durum Bernie için neden farklı olsun? Bu fikrin ardındaki neden ~A’nın rasyonel olmayan bir şekilde edinilmiş bir inanç olması gibi görünüyor. Öyleyse Bernie’nin yeniden Tanrı’ya inanmaya başlamasında işi yapan gerçekten ontolojik argüman mı yoksa basitçe ~A’nın rasyonel olmayarak edinilmiş olması mıdır? Eğer o ~A veya B’ye inanmayı bırakması gerektiğine ve inanmayı bırakması gereken inancın rasyonel olmayan nedenlerle edinildiği için ~A olduğuna ikna olduysa A’ya inanmaya direkt olarak ~A’ya inancının temellerinin rasyonel olmadığını görme yoluyla da ikna olacaktır. Dolayısıyla öyle görünüyor ki çatışan inançlarından birini bırakması gerektiğini söyleyen ilk parça Bernie’nin teizme ikna olmasında bir rol oynamıyor. Bernie C’ye veya ontolojik argümana ihtiyaç duymaksızın teizme döner. Esasen Johnson’ın argümanı ilahi his gibi dini tecrübeye de dayanabilir. Bernie geçmişte yaşadığı bir dini tecrübe dolayısıyla A’ya rasyonel olarak inanıyorken tecrübesini unutması veya bir başka nedenle reddetmesi dolayısıyla A için temelini kaybettiyse ve zamanla A’ya inanmayı da böylece rasyonel olmadan bıraktıysa dini tecrübesini hatırladığında veya kabul ettiğinde zaten ontolojik argümana dayanmaksızın A’ya inanmaya yeniden başlayacaktır.

2.5. Motivasyonel Olarak Merkezi İnançlardan Yola Çıkan Argüman

Pruss’un imkan öncülü lehine sezgilere hitap eden, bir o kadar da orijinal bir argümanı daha vardır8. Argümanını şöyle förmülize etmiştir:

  • (5) x, motivasyonel olarak merkezi bir p inancına sahipken önemli bir zaman boyunca, serpilmiş ve entelektüel olarak sofistike yaşamlar süren bir birey veya topluluk ise muhtemelen p mümkündür.
  • (6) Bir miktar birey ve topluluk maksimum mükemmellikte bir varlığın var olduğuna dair motivasyonel olarak merkezi inanca sahipken önemli bir zaman boyunca, serpilmiş ve entelektüel olarak sofistike yaşamlar sürmüştür.
  • (7) Öyleyse muhtemelen maksimum mükemmellikte bir varlık mümkündür.

Argüman hakkında söylenmesi gereken ilk şey, diğer argümanlardan farklı olarak imkan öncülünü muhtemel kılma iddiasında olduğudur. İkincisi, bu muhtemellik Pruss’un da belirttiği gibi tüm koşullar değerlendirilmiş bir muhtemellik değil diğer her şey eşit tutulduğunda söz konusu olan bir tür muhtemelliktir.

Motivasyonel olarak merkezi inançlar ile neyin kastedildiğini anlamak önem taşıyor. Bunun için Pruss’un kavramı açıklamak için kullandığı örneklere bakabiliriz. Sahip olduğumuz inançların motivasyonel merkezilikleri hiyerarşik ilişkiye sokulabilir. Bir kimsenin M13 küresel yıldız kümesinin çoğunlukla yaşlı yıldızlardan oluştuğuna dair inancı farklı olsaydı, söz gelimi bu kişi M13’ün ağırlıkla genç yıldızlardan oluştuğuna inansaydı hayatı önemli ölçüde değişmezdi. Diğer taraftan bir kimsenin evli ve çocuk sahibi olduğuna dair inancı hayatını ve eylemlerini önemli ölçüde etkiler konumdadır. Bu inancın olmaması durumunda hayat bambaşka bir şekil alacaktır. Bu açıdan bakıldığında bir kimsenin evli ve çocuk sahibi olduğuna dair inancının motivasyonel olarak merkezi olduğu söylenebilir. Şüphesiz bu iki inanç arasında bir yerlerde bulunan inançlar da vardır. Örneğin diş fırçalamanın sağlıklı olduğuna dair inanç, M13 küresel yıldız kümesine dair inançtan motivasyonel olarak daha merkeziyken aileye dair inançtan daha merkezi değildir.

İnançlar merkezi ancak yanlış olabilir. Hatta, Pruss’a göre, merkezi ancak yanlış bir inanç serpilmiş ve entelektüel olarak sofistike bir hayatı destekleyebilir de. Her ne kadar etkili bir tedavi metodu olduğuna dair araştırmalar aslında hatalı olsa da bir kanser tedavi metodu üzerinde çalışan bir doktoru örnek olarak verebiliriz. Diğer taraftan, serpilmiş ve entelektüel olarak sofistike bir yaşama katkı sağlayan merkezi bir inancın –yanlış olmasının ihtimal dahilinde oluşuna karşın- imkansız olması oldukça ihtimal dışıdır. Argümanının ilk öncülünü ifade eden bu nokta için Pruss iki argüman öne sürer.

Birincisi, imkansız bir önerme tüm önermeleri gerektirir. Birey ve topluluklar motivasyonel olarak merkezi inançlarının gerektirdiği diğer inançlara da uygun yaşamlar süreceğine göre serpilmeleri ve mutlulukları için iyi olmayan inançları benimseme riskiyle karşı karşıyadırlar. Örneğin yuvarlak karelere merkezi biçimde inanan bir birey veya topluluk, bu inançlarının bir sonucu olarak acının mutluluk getirdiğine de inanabilir ve böylece insanlara mutluluk vermek için işkence edebilir ki bu onların hayatına ciddi biçimde zarar verir.

Bu argüman ilk bakışta sağduyuya uygun görünmüyor. Burada kullanıldığı anlamıyla “gerektirme” öncülü destekleyecek bir zemin oluşturmaktan uzak olabilir zira örneğin bir kimsenin evli ve çocuk sahibi olduğuna dair inancı Fermat’ın son teoremini de gerektirir (entail) ancak aklı başında hiçbir birey veya topluluk ilk inançtan teoremi çıkarmaya (derrive) kalkmayacaktır. Paralel olarak yuvarlak karelere inanan bir topluluk da bu inançtan acının mutluluk getirdiğine dair inancı çıkarmayacaktır. Ek olarak, böyle çıkarımlar yapılacağını kabul etsek dahi birey ve topluluklar olarak inançlarımızı tetkik ettiğimiz için çıkardığımız yanlış inançları eleyeceğimiz, dolayısıyla örneğin yuvarlak karelerden acının mutluluk getirdiğine dair bir inanç çıkardığımızda çıkarılan inancın bariz yanlışlığından dolayı bir an için duraksayıp çıkarımımızda bir hata olduğunu düşüneceğimiz söylenebilir.

İkinci argümana göre ise serpilmiş ve entelektüel olarak sofistike yaşamlar iyi değerlendirilmiş yaşamlardır ve yaşam için iyi bir değerlendirme, inançların tutarlılığını iyi bir şekilde değerlendirmeyi de gerektirir. Argümanın temel güçlüğü, maksimum derecede mükemmel bir varlık gibi kompleks kavramların iyi değerlendirilmesinin kolay olmamasıdır. Örneğin insanların büyük bir kısmı, değerlendirilmeleri profesyonel bir kavrayış  gerektirebilen tanrısal sıfatların tutarlılığı tartışmalarından haberdar bile değildir. Bu noktada imkan öncülü lehine tasavvur edilebilirlik argümanındaki prima facie ve ideal tasavvur edilebilirlik arasındaki  ayrıma benzer bir durum olduğu düşünülebilir.

Pruss’un öne sürdüğü iki argümanın ikna ediciliği yüksek olmasa da ilk öncülün kendi başına da makul göründüğü veya lehine başka argümanlar da geliştirilebileceği söylenebilir. Ancak argümana daha ciddi bir eleştiri, benzer şekilde Tanrı’nın var olmamasının mümkün olduğunu destekleyecek şekilde de kurulabileceğini göstermekle olacaktır. Pruss ikinci öncül için önemli dindar isimler ve dini topluluklardan örnekler verir. Buna karşılık olarak da nonteist birey ve topluluklardan söz edilebilir. Pruss bu tarz bir itiraza cevap vermenin üç yolu olduğunu belirtir: (a) söz konusu inanç aslında teizmle çelişmez, (b) inanç serpilmiş ve entelektüel olarak sofistike bir yaşam için merkezi değildir veya böyle bir yaşama teizmden daha az rasyonel katkı sağlar, (c) inanç ikna edici bir argümanla yanlışlanabilir.

Her koşulun değerlendirildiği değil tüm ek koşulların ihmal edildiği bir tür muhtemellik tartışıldığına göre (c) açık bir yöntem değildir. (a) ise her ne kadar Pruss savunusunu bir miktar yapsa da geniş bir kullanıma açık değildir zira maksimum mükemmellikte bir varlığın var olmamasının mümkün oluşunu gerektiren herhangi bir inanç  basitçe teizmle çelişecektir. Pruss, incelediği politeizm, panteizm, illüzyonizm ve ateizm örneklerinde politeizm ve panteizmin en azından bazı türlerinin teizmle çelişmediğini göstermeye çalışır. Ancak odaklandığı, detaylarda kaybolmamak adına değinmeyeceğimiz noktalar hatalıdır. Politeizmin teizmle çelişmesinin temel nedeni, birden fazla ibadete layık varlığı içermesidir. Panteizme göre ise Tanrı’nın kişisel olmayan, fiziksel parçalardan oluşan bir varlık olması mümkündür, dolayısıyla da panteizm teizmle çelişir.

Daha geniş bir kullanıma açık olan ve tartışmanın temelini teşkil eden yolun (b) olduğunu söyleyebiliriz. Ancak (b) üzerinden dönecek tartışmayı, tartışılan inancın merkezi olmadığıyla veya hayata rasyonel katkısının azlığıyla sınırlandırmamalıyız, Pruss’un başka bir yerde belirttiği üzere argümana konu olan muhtemelliğin boyutunu belirleyen birçok faktörden bahsedilebilir. Verdiği örnekleri üç başlık altında gruplandırmak gerekirse bunlar şu şekildedir:

  • İnancın merkeziliği ile ilgili faktörler: inancın ne kadar merkezi olduğu, x’in serpilmiş yaşamına dair eylemlerin ne kadarının söz konusu inançtan kaynaklandığı, inanç ile bu eylemler arasındaki bağın ne kadar rasyonel olduğu.
  • Birey ve topluluk yaşamlarının niteliksel değeri: x birey veya toplumunun ne kadar serpilmiş olduğu, entelektüel olarak ne kadar sofistike olduğu.
  • Birey ve topluluk yaşamlarının niceliksel değeri: ne kadarlık bir zaman aralığının söz konusu olduğu, topluluğun ne kadar büyük olduğu.

Bu kriterler altında maksimum mükemmelliğin var olmasını ve olmamasını içeren inançlar farklı kriterlerde birbirine üstünlük sağlayabilir. Sonucu, hangisinin mümkün olmasının daha muhtemel olduğunu belirleyecek olan ise toplamda nasıl bir yerde olunduğudur. Buradaki amacımız açısından ise sonucun maksimum mükemmellik lehine olmadığını göstermemiz yeterli olacaktır. Maksimum mükemmelliğin var olduğuna dair inançlara kısaca teizm, var olmadığına dair inançlara ise yine kısaca nonteizm diyeceğiz.

Tersten başlayalım. Birey ve topluluk yaşamlarının niceliksel değeri bakımından teizm ile nonteizm arasında nasıl bir fark vardır? Antik Çin, Antik Hindistan ve Antik Yunan gibi büyük medeniyetlerde nonteist inançların hakim olduğuna dikkat çekerek başlayabiliriz. Bu medeniyetlerde yukarıda sözünü ettiğimiz politeist, panteist ve Pruss’un illüzyonist olarak adlandırdığı inançlar vardır. İnsanlık tarihinin en yaşlı dinlerinin önemli bir kısmı da nonteizm çatısı altına dahil edilebilir: Hinduizm, Jainizm, Konfüçyüsçülük, Budizm, Taozim, Şintoizm… Günümüzde de teizmin en büyük temsilcileri olan Hristiyanlık ve İslam’ın toplam inanan sayısı dünya nüfusunun yaklaşık olarak yarısını teşkil ediyor. Ancak her iki dinin de insanlık tarihinde yeni olduğu da akılda tutulmalıdır. Tüm bunlara dayanarak gerek benimsendikleri zamanın büyüklüğü gerekse benimseyen birey ve toplulukların çokluğu bakımından nonteistik inançların teistik inançlara üstünlük sağladığını ya da daha küçük bir savda bulunarak teizmin nonteizme üstünlük sağlayamadığını iddia edebiliriz.

Birey ve topluluk yaşamlarına niteliksel olarak bakacak olursak ne söyleyebiliriz? Öncelikle bir noktayı vurgulamalıyız. Birey ve toplulukların ne kadar gelişmiş yaşamlara sahip olduklarını tartışırken gelişimin neye endeksli olduğuna dikkat etmek önemlidir. Söz gelimi ibadete adanmış bir ömür, serpilmiş bir yaşam örneği olarak bir ateiste tatsız görünecekken sadece farklı dünyevi hazlarla dolu bir yaşam da bir teisti ikna etmeyecektir. Hayatı değerli kılan unsurlara ilişkin anlayışlardaki bu farklılıklar farklı dinler söz konusu olduğunda daha da derinleşebilmektedir. Örneğin dinlerden biri Tanrı uğruna savaşmayı yüceltirken bir diğeri öldürmeyi her koşulda yasaklayabilir. Farklı inanç sistemlerinde değerli hayatlar yaşamaya dair farklı anlayışlar olduğuna göre ise bu başlık altındaki faktörler ile teizm ve nonteizmin birbirine üstünlük sağlaması diğerlerine kıyasla daha zordur. Örneğin Pruss’un, dış dünyanın bir illüzyondan ibaret olduğunu ifade eden, ağırlıkla Doğu medeniyetlerinde yer bulan illüzyonist inançların, gelişmiş değerli yaşamlar için olmazsa olmaz olan başkalarına karşı sevgiyi dışlamak zorunda olduğuna dair iddiası bu anlamda iki sebeple etkisiz kalabilir. Birincisi, tahsilli bir illüzyonist, bu iddianın yanlış olduğuna inanmak için güçlü gerekçelere sahip olabilir. İkincisi, iddia doğruysa dahi Pruss’un anladığı anlamıyla sevginin serpilmiş ve değerli bir yaşam için merkeziliği bir illüzyonist için o kadar da merkezi olmak zorunda değildir; illüzyonist, inancı gereği başka daha büyük değerlere sahip olabilir. Bu ayrım için söylenebilecek son bir söz, yukarıda da bahsettiğimiz medeniyet ve inançlar altında milyarlarca kişinin binlerce yıldır, kendi inanç sistemlerine endeksle, serpilmiş ve değerli yaşamlar sürmüş ve sürüyor olduklarıdır.

Tartışmanın ortak bir zeminde yürütülebilir, bir yaşamı serpilmiş ve değerli kılan unsurlar konusunda uzlaşı içinde olunabilecek bölümü içinse biz de Pruss gibi örneklere başvurabiliriz. Hypatia Bradlaugh Bonner, Jeremy Bentham, Çarvaka Okulu ve Ajita Kesakambali, el-Maarrî, Christine Overall, Graham Oppy, Willard Van Orman Quine, Gautama Buddha, Lucretius ve daha nicesi (kimilerinin dini görüşlerinden çağlarının koşulları veya haklarındaki bilgi eksikliğimiz dolayısıyla emin olmak zor olsa da) nonteist inançlarla serpilmiş, değerli ve entelektüel olarak sofistike yaşamlar sürmüştür. İlginç bir not olarak,  Pruss’un argümanı gereği burada zihinlerinde mutlak güçlü, bilgili ve iyi zorunlu bir varlıktan daha farklı birer Tanrı tasavvuru olanlar da tanım gereği maksimum mükemmelliğin var olmadığını düşündükleri için nonteizme dahil edilecektir. Örneğin Tanrı’nın zorunlu olarak var olmadığına inanan Richard Swinburne, talihsizce, yukarıda sayılanlarla birlikte nonteist sınıfını destekleyen bir örnek olarak karşımıza çıkıyor. Önemli bir diğer nokta da kendi yaşamlarının tutarlılığını ve inançları eleştirel olarak incelemeye muhtemelen en müsait grup olan felsefecilerin çoğunluğunun nonteist olmasıdır. Din felsefecilerinde bu çoğunluğun teistlere kayması ise bir nedenle örneği teizm lehine çevirmez: teistler, nonteistlerin din felsefesi çalışmak için sahip olduğu tüm nedenlere ek olarak onlar için çok önemli başka nedenlere de sahiptir zira varlığına inanan biri için Tanrı muhtemelen her şeyden önemli olacaktır. Birey ve topluluklara ek olarak bireylerin ve insanlığın gelişimine ciddi katkılar sağlayan ideoloji ve inançlardan da örnekler verebiliriz. Natüralizm, bilimin gelişiminde çok ciddi bir pay sahibidir. Seküler hümanizmin de birey ve toplumlar üzerinde insana bakış,  ahlak, hukuk gibi alanlarda ciddi etkileri vardır.  

Son olarak, teistik ve nonteistik inançların merkeziliğini inceleyelim. Nonteistik inançların merkeziliği ile ilgili zikredilmesi gereken muhtemelen ilk nokta nonteist başlığı altına dahil ettiğimiz inançların, Pruss’un negatif bir doktrin olarak nitelediği ateizmle sınırlı olmamasıdır. İnsanlık tarihinde kendine yer bulmuş din, ideoloji veya inançlardan evrenin kişisel; mutlak güçlü, iyi ve bilgili; tek ve zorunlu bir yaratıcısı olduğunu reddedenlerin tümünü bu başlığa dahil ediyoruz. Dolayısıyla bu anlamıyla insanlık tarihinde birçok, hatta teist dinlerden daha fazla nonteist din, ideoloji veya inanç mevcuttur. Yukarıda bahsi geçen dinler örnek olarak verilebilir. Bu dinlerden herhangi birisinin tek tanrılı dinlerden daha az merkezi olduğunu iddia etmekse güçtür. Nihayetinde ibadet, evrene dair dinin penceresinden bir kavrayış veya belirli bir ahlak anlayışı gibi tek tanrılı dinlerin, inananlarının hayatını şekillendirdiğinin iddia edilebileceği noktalar ağırlıkla bu dinlerde de bulunmaktadır.

Dinler haricinde bir örnek natüralizmden verilebilir. Modern bilimin natüralist paradigma ile yükselmesi, entelektüel serpilmeye güzel bir örnektir. Ne var ki Pruss, bunun argümanla ilişkisi hakkında kuşkucudur. Ona göre natüralizm nedenselliğin yalnızca doğal nedensellik olduğunu ifade ettiğine göre natüralist bir bilim insanının inancı aslında nedenselliğin olmaması ile de uyumludur, dolayısıyla natüralizmden evrenin ampirik araştırmayla anlaşılabileceği inancına geçiş zayıftır. Bu noktada serpilmeyi sağlayanın birçok örnekte tekil inançlardan ziyade inanç kümeleri olduğunu gözlemlememiz gerekiyor. Bir dindar serpilmesinin önemli bir bölümünü maksimum mükemmelliğin örneklendiğine inancı ile doğrudan ilişkisi olmayan başka inançlarına borçludur. Örneğin dindarlar teist bir dini benimser, dinleri -birer inanç kümesi olarak- maksimum mükemmelliğin örneklenebilir (genelde örneklenmiş) olmasını gerektiren parçalara sahiptir. Benzer şekilde natüralist bilim insanları da natüralizmi, doğal fenomenlerin açıklamalara sahip olduğuna (Pruss’un iddia ettiğinin aksine bunun yeter sebep ilkesi ile özdeş olması gerekmiyor) ve bunların ampirik araştırmalarla anlaşılabileceğine inançları içeren inanç kümelerine sahiptir ve serpilmeyle ilişkili olan, din örneğindeki gibi bu kümedir. Bu inanç kümesi ise natüralizmi içerir ve dolayısıyla nonteizmi gerektirir. Sonuç olarak bilim insanları ve natüralizm hakkındaki akıl yürütme sorunluysa teist dinler hakkındaki akıl yürütme de aynı sorunu paylaşır.

Peki farklı nonteist inançların ardından ateizm için ne söylenebilir? İlk olarak Pruss’un teist dinlerin karşısına çekirdek anlamıyla ateizmi koyma tercihine bir miktar baskı uygulayıp bir ayrımı fark edelim. Bir önceki paragrafta da değinildiği gibi maksimum mükemmelliğe inançları sayesinde serpilen birey ve topluluklar için serpilmeyi getiren çekirdek anlamıyla teizm değil büyük oranda teizmi içeren inanç sistemleridir. Yalnızca kadir-i mutlak, alim-i mutlak ve ahlaken kusursuz bir varlığa inanç, örneğin bu varlığın vahiy yoluyla gönderdiği mesajlar olmaksızın ne tür bir serpilmeye öncülük edebilir? Eğer Pruss argümanını desteklemek için teist inanç sistemlerini kullanacaksa bunun karşı kamptaki muadili basitçe Tanrı’nın var olmadığını ifade eden çekirdek anlamıyla ateizm değil ateist inanç sistemleridir -bahsi geçen ateist din ve ideolojiler gibi. Karşılaştırma için çekirdek anlamıyla ateizm tercih edilecekse de argümanın destek alacağı örnekler yine çekirdek anlamıyla teizm üzerinden olmalıdır ancak bu anlamlarıyla değerlendirildiklerinde teizmin ateizmden motivasyonel olarak daha merkezi olduğu hiç de açık değildir.

Ateist inanç sistemleri halihazırda incelemiş olduğumuz nonteist din ve ideolojilere dahil olduğu için ikinci yolun izlendiğini varsayalım ve ateizmin çekirdek anlamıyla serpilmiş yaşamlar için merkeziliğini sorgulayalım. Şüphesiz, bir ateistin ateizmini bırakması yaşamının önemli ölçüde değişmesini beraberinde getirecektir. Ateist inancın merkeziliği birkaç açıdan incelenebilir. Bir ateist, zorunlu olarak olmasa bile, muhtemelen ahirete inanmayacaktır. Bunun -kimi teistlerce de eleştiri odağı yapıldığı üzere- dünya hayatına ve içerisindeki dünyevi unsurlara bir teistten çok daha fazla önem vermesini beraberinde getirmesi de doğaldır. Örneğin böylece bir ateist için mümkün olduğunca fazla deneyim yaşamak daha cazip görünebilir. Ahiretle de bir yere kadar ilişkili bir diğer husus ilahi adalettir. Bir ateistin yine zorunlu olmasa da muhtemelen karma gibi bir inanca sahip olmayacağını söyleyebiliyorsak onun için eninde sonunda hak yerini bulmak zorunda değildir, benzer şekilde her şerde bir hayır da yoktur. Bunun da gerek bireylerin günlük eylemlerinde ve sorunlara karşı aldığı tavırlarda, gerek toplumsal hareketler üzerinde etkisi olacağı söylenebilir. Bunlara ek olarak bir ateist, yine muhtemelen, seküler bir ahlak anlayışına sahip olacaktır. Bu şekilde eşcinsel birliktelik, zina, kürtaj, veganizm veya faiz gibi günlük hayatın içinde olan, örnekleri uzunca artırılabilecek tartışmalarda din temelli argümanların onun nezdinde bir ağırlığı olmayacaktır. Son olarak bir ateist eşini Tanrı’nın sevgisinin bir yansıması olduğuna inandığı için değil, muhtemelen yalnızca eşine duyduğu sevgiden dolayı öper; ihtiyaç sahiplerine onların Tanrı’nın evlatları olduğuna inandığı için değil, yalnızca ihtiyaç sahibi oldukları için yardım eder. Pruss tersini daha değerli görüyor olsa da şeylere birincil olarak başka bir varlığa referanslı nedenlerle değil yalnızca kendileri için değer veriyor olmak bir ateiste daha değerli gelebilecektir. Şeylere sırf kendileri için değer verilmesinin örnekleri de artırılabilir. Detaylandırılmaya ihtiyaç duysa da konudan sapmama ve yer kaygılarıyla daha fazla derinleştiremeyeceğimiz bu nedenlerle ateizmin de hatırı sayılır ölçüde merkezi bir inanç olduğuna ve serpilen yaşamlara eşlik ettiğine inanıyoruz.

Tüm bunlar ışığında söyleyebiliriz ki inancın merkeziliği için teist inançlar; birçok din, ideoloji, felsefi pozisyon içeren nonteist inançlara belli noktalarda üstünlük sağlayabilirken belli başka noktalarda da onların altında kalmaktadır. Böylece inancın merkeziliği faktöründe teizmin net bir üstünlük sağladığından güçlü ölçüde şüphe edilebilir. Yaşamların niteliksel faktörlerinde de özellikle niteliksel değerin inanca endeksli olmasından ötürü tarafların birbirini Pruss’un argümanı ile ikna etmesi zordur. Ek olarak teist örneklere paralel nonteist örnekler de vardır. Dolayısıyla bu faktörde de teizmin net üstünlüğü söz konusu değildir. Nicelik faktörü içinse nonteizmin önde olması daha olasıdır. Faktörlerden hangilerinin diğer her şey ihmal edildiğindeki muhtemellikte daha büyük ağırlığa sahip olduğu da tartışılabilecek olsa da son tahlilde teizmin nonteizmden daha üstün bir konumda olduğunu reddetmek için ciddi sebeplerimiz olduğu görülüyor ve bu noktaya varmak amacımız açısından yeterlidir. Pruss’un argümanı, rakip iki ontolojik argüman arasındaki simetriyi bozmak için yetersizdir.

2.6. Maksimal Tanrı Argümanı9

Yujin Nagasawa, imkân öncülü lehine halihazırda öne sürülmüş argümanları iki aşamalı olarak değerlendirir. x; “tasavvur edilebilir”, “deneyimlenebilir”, “deontik olarak mükemmel” veya “motivasyonel olarak merkezi bir inancın nesnesi” olmak üzere

  • I. Tanrı’nın x olduğunu göster.
  • II. x’in imkanı gerektirdiğini göster.

Ancak ona göre argümanlar bu iki aşamadan en az birinde sınıfta kalmaktadır. Kendi argümanının ise tek bir hamleyle imkan öncülünü gösterdiğini ifade eder. Bunun nasıl olduğunu anlamak için geliştirdiği Maksimal Tanrı anlayışına bakmalıyız.

Nagasawa klasik kadir-i mulak, alim-i mulak ve ahlaken kusursuz Mutlak Tanrı (omni-God) anlayışı ile kendisinin Maksimal Tanrı dediği modelleri ayırır. İlkine göre Tanrı mükemmel yapıcı niteliklere maksimum düzeyde sahipken ikincisinde maksimum mümkün kombinasyonda sahiptir. Örneğin mutlak kudret, Tanrı tarafından yapılamayacak şeyler olduğu için Tanrı’nın bir sıfatı olmayabilir. Tanrı’nın mutlak iyiliği kötülük yapmasını engellediği için de aynı şekilde kudreti sınırlı olabilir. Bu durumda Tanrı’nın bu mükemmel yapıcı niteliklere tek tek maksimum seviyede değil ancak kendi içerisinde ve birbiriyle tutarlı olacak şekilde toplamda maksimumu vermek üzere sahip olduğunu söyleyebiliriz. Nagasawa’ya göre Anselmci mükemmel varlık teizmi Mutlak Tanrı modelini zorunlu olarak gerektirmez. Mükemmel varlık teizmi Maksimal Tanrı modelini gerektirir ve Maksimal Tanrı modeli Mutlak Tanrı modeliyle uyumludur. Eğer mükemmel yapıcı nitelikler kendi içlerinde ve birbiriyle tutarlıysa Maksimal Tanrı modeli Mutlak Tanrı modeline eşit olacaktır.

Nagasawa’nın tezi verildiğinde, imkân öncülü lehine Maksimal Tanrı argümanı, Nagasawa’nın da kullandığı tabirle ironik bir şekilde, aslında bir argümana ihtiyacımız olmadığını ifade etmektedir. Maksimal Tanrı yorumuna göre Tanrı mükemmel yapıcı niteliklerin tutarlı maksimum kombinasyonuna sahip olduğuna göre otomatik olarak imkan öncülünü kabul edebiliriz. Üstelik bu, yukarıda bahsi geçen argümanların aksine imkan öncülüne tek bir hamleyle işaret eder.

Hayranlık uyandırıcı bir tez ve argüman olmasına karşın Nagasawa’nın Tanrı anlayışının imkan öncülünü direkt olarak göstermediğini söyleyebiliriz. Her ne kadar o argümanını tek aşamalı olarak görse de aslında değindikleri gibi iki aşamalı bir akıl yürütmeye dayanır:

  • (8) Tanrı tutarlıdır.
  • (9) Tutarlılık imkanı gerektirir.
  • (10) Dolayısıyla Tanrı mümkündür.

Maksimal Tanrı tezinin ilk aşamayı garanti ettiğini belirten Nagasawa, muhtemelen tutarlılığın otomatik olarak imkanı beraberinde getireceğini düşündüğü için ikinci aşamayı/öncülü anlamsız bulmuştur. Bu nedenle ikinci aşamayı gerekçelendirmese de biz imkanın tutarlılık ile garanti edilemeyeceğini savunabiliriz. İlk bölümdeki içsel tutarlılık – metafiziksel imkan ayrımlarını ve imkan öncülü lehine bazı argümanların bu ayrımıyla ilgili olarak değindiğimiz kusurlarının bir örneği burada da karşımıza çıkıyor. Modal ontolojik argümanın çalışması için ihtiyaç duyulan öncül Tanrı’nın metafiziksel olarak mümkün olduğudur, nitelikleri arasında bir çelişki olmaması veya mantıksal olarak mümkün olması değil. Bir varlığın mantıksal olarak mümkün olması, mantık yasalarını ihlal etmediği anlamına gelirken metafiziksel olarak mümkün olması, onun var olabileceği anlamına gelir. Bunların neden farklı olduğunu görmek adına, bir önermenin doğru olabilmesi için karşılaması gereken şartları inceleyebiliriz. Diyelim ki bir p önermesi için onun reddini zorunlu olarak gerektiren bir q önermesi vardır. Bir diğer ifadeyle <q ise ~p> tüm mümkün dünyalarda doğrudur. Öyleyse p’nin mümkün olması için şu iki şart zorunlu olmalıdır (yeter olmayabilir): (i) p tutarlıdır, (ii) q tüm mümkün dünyalarda doğru değildir. Somutlamak adına yine örneğimizi düşünelim. Gereksiz kötülük -eğer gereksiz kötülüğün böyle olmadığını düşünüyorsanız başka bir örnek de düşünebilirsiniz- gibi Tanrı’nın varlığıyla çelişen bir durumun tutarlı olduğunu kabul edelim. Bu durumda gereksiz kötülüğün var olduğu mümkün dünyaları yine de garanti edemeyiz zira eğer tüm mümkün dünyalarda Tanrı varsa gereksiz kötülük imkan dahilinde olmayacaktır. 

p’nin mümkün olması için zorunlu olduğunu öne sürdüğümüz iki şartı, p’nin olumsal olmayan bir önerme olduğu bir durum için güncelleyelim -yeniden, öne sürdüklerimizin yeter şart olduğunu söylemiyoruz. (i) p tutarlıdır, (ii*) q’nun doğru olduğu mümkün bir dünya yoktur. Bunu da örneği tersten düşünerek somut hale getirebiliriz. Tanrı içsel olarak tutarlı olsa dahi gereksiz kötülüğün varlığı mümkünse hiçbir mümkün dünyada yoktur, imkansızdır. Sonuç olarak söyleyebiliriz ki Nagasawa’nın öne sürdüğü tarzda bir argüman tek aşamalı olamaz. İçsel tutarlılığın imkanı gerektirdiği açık değildir, hatta savunusunu yaptığımız üzere yanlıştır. Dolayısıyla Tanrı’nın tutarlılığını göstermek bir paket halinde imkan öncülünü de beraberinde getirmez. Yine de bir varlığın tutarlılığını göstermenin onun mümkün oluşunu garanti etmese de bu fikri destekleyeceği söylenebilir. Ancak bu kabul edilse dahi Nagasawa’nın tezi ciddi bir ilerleme sayılmaz zira gereksiz kötülük gibi Tanrı’yla bir arada var olamayacak durumların içsel tutarlılığı konusunda da çok ciddi ve yaygın şüpheler yoktur.

Belki de daha ilginci, Nagasawa’nın argümanında ilk aşama, Tanrı’nın tutarlı olduğunu gösterme adımı da yeterince yerine getirilmiş görünmüyor. Nagasawa’nın Maksimal Tanrı tezini ortaya koyarken karşılamayı hedeflediği argüman türlerini hatırlayalım. Bu argümanlar tanrısal sıfatların bireysel olarak ya da iki veya daha fazla üye içeren gruplar halinde tutarsız olduğunu öne sürme gibi bir stratejiye dayanıyordu. Bu tip stratejilere dayanan argümanların yeni bir Tanrı anlayışıyla iptal edilmesi, hala ayakta kalan epistemik olarak mümkün senaryoların bulunması nedeniyle Tanrı’nın içsel olarak tutarlı olduğunu direkt olarak göstermez. Bu senaryolardan ilkine göre (a) bir maksimum mükemmellik noktası olmayabilir. Nicel olarak sonsuza dek artırılabilen mükemmel yapıcı nitelikler varsa ve bu yolla her varlık için kendisinden daha üstünü mümkünse, mümkün varlıkların en mükemmeli diye bir kavram anlamsız olacaktır. Örneğin diyelim ki ne kadar bilgili/güçlü/iyi olursa olsun her mümkün varlık için ondan daha bilgilisi/güçlüsü/iyisi de mümkündür. Bu durumda mükemmel yapıcı niteliklerin birbiriyle tutarlı biçimde maksimum düzeyde örneklenmesi mümkün olmazdı. Burada önemli olan bir husus, mükemmel yapıcı niteliklerin birer üst sınırı olmamasına yönelik bunun gibi ihtimallerin Nagasawa’nın muhaliflerince temellendirilmesi gerekmez. Bir maksimum mükemmellik noktası olduğunu gösterme iddiasında olduğuna göre kanıt yükümlülüğü Nagasawa’dadır. Öte yandan üst sınırın olmadığına dair bu tarz ihtimallerin temellendirilmesi durumunda varılacak sonuç zaten modal ontolojik argümandan bağımsız olarak Tanrı’nın var olamayacağı olurdu.

Bir diğer senaryoya göre bir maksimum mükemmellik düzeyinden söz edebiliyor isek dahi (b) bu düzeyin örneklenebileceği birden fazla yol olabilir. Maksimal Tanrı teziyle birlikte kadir-i mutlak, alim-i mutlak ve ahlaken kusursuz Tanrı modelini, tanrısal sıfatlar için epistemik olarak tek senaryo olmaktan çıkardığımıza göre böylece maksimum mükemmellik düzeyine ulaşmanın birden fazla yolunun olması olasılığını da teslim etmeliyiz. Güç, bilgi ve iyiliğin kendi başlarına sahip olunabilecekleri maksimum düzeyin 10 birim ile ifade edilebildiğini farz edelim. Ancak yine diyelim ki bu nitelikler arasındaki çatışmalardan dolayı üç niteliğe de 10 birim değerinde sahip olmak mümkün olmasın ancak 9 birim güç, 10 birim bilgi, 10 birim iyilik ve 10 birim güç, 10 birim bilgi, 9 birim iyilik bir varlığın maksimal anlamda mükemmel olması için bu niteliklere aynı anda sahip olabileceği iki yol olsun. Böyle bir durumun  -ne kadar olduğundan bağımsız olarak- epistemik açıdan ihtimal dahilinde olduğu söylenebilir. Ancak bu tarz bir senaryo Tanrı’nın bu üç niteliğe hangi gruptaki gibi sahip olduğunun açıklamasını gerektirmesi bakımından teizm için sorunludur. Yeniden, burada Nagasawa’nın Tanrı ve mükemmel yapıcı niteliklere getirdiği yeni yaklaşımın Tanrı’nın tutarlılığını direkt olarak göstermediğini ortaya koymayı amaçladığımız için bunun gibi bir senaryonun ne kadar ihtimal dahilinde olduğu birincil önem taşımaz. 

Son olarak bir senaryo da ibadete layık olma ile ilgili olabilir. Maksimum mükemmellik düzeyinin örneklenmesinin tek bir yolu olsa dahi Maksimal Tanrı anlayışı ile birlikte Tanrı’dan daha güçlü, bilgili veya iyi varlıkların mümkün olması ihtimaline kapı araladığımız için (c) maksimum derece mükemmel varlık ibadete layık olmayabilir. Bu senaryo –yer kaygısıyla, ibadete layık olma gibi birçok eksende tartışılabilecek bir sıfat hakkındaki tartışmalardan kaçınıp problemi başlık aşamasında sunmak durumundayız- ibadete layıklığın daha güçlüsü, bilgilisi, iyisi mümkün olmama şartlarını gerektirdiği sezgisine dayanır. Örneğin, her ne kadar kendisinden nihaî olarak daha mükemmeli mümkün olmasa da maksimum derecede mükemmel bir varlıktan ahlaken daha iyi bir varlığın mümkün olmasının, söz konusu maksimum mükemmellikteki varlığın ibadete layıklığını şüpheli hale getireceği düşünebilir. Ancak ibadete layık olmak, muhtemelen herkesin üzerinde uzlaşacağı üzere Tanrı tanımının olmazsa olmaz bir parçası olduğuna göre bu ciddi bir sorundur. Sonuç olarak Nagasawa’nın argümanı tek değil, eleştirdiği argümanlar gibi iki aşamalıdır ve iki aşamadan da şüphe etmek için ciddi zeminler vardır.

2.7. Değer Argümanı

Joshua Rasmussen, ters modal ontolojik argümanın imkan öncülünü destekleyen argümanlardan birinde bir benzerini kullanacağımız bir akıl yürütmeyle, imkan öncülü lehine yaratıcı bir argüman geliştirmiştir.10

  • (11) Bazı miktarlarda değer örneklenebilir.
  • (12) Bazı miktarlarda değer örneklenebilirse, tüm miktarlarda değer örneklenebilir.
  • (13) Dolayısıyla, tüm miktarlarda değer örneklenebilir.
  • (14) Maksimum derecede mükemmellik, değerin bir miktarıdır.
  • (15) Dolayısıyla maksimum derecede mükemmellik örneklenebilir.

Maksimum derecede mükemmellik veya Tanrı’yı mümkün olan en değerli varlık olarak ele aldığımızda üç öncüle sahip argüman geçerlidir. Rasmussen’in de ortaya koyduğu gibi ilk öncülü, bazı miktarlarda değerin örneklenebilir olduğunu, kabul etmek için belki de en iyi gerekçemiz bazı miktarlarda değerin örneklenmiş olduğudur. Örneğin siz başka herhangi bir insan gibi önemli ölçüde değer teşkil etmektesiniz. İkinci öncülse salt nicelikle ayrılan durumların imkan bakımından ayrılmayacağına dair “vazgeçilebilir” ilkeye –modal devamlılık ilkesi- dayanır. Salt nicelik bakımından ayrılan önermelerin ya tümü mümkündür ya da tümü imkansızdır/zorunludur. Değerin bazı miktarları mümkün olduğuna göre tüm miktarları öyle olmalıdır.

Son öncül olan (14) içinse Rasmussen’in argümanı basittir: maksimum mükemmellik muayyen (determinate) bir miktar değerdir -maksimum değer-; muayyen miktarlardaki tüm değerler değerin bir miktarıdır; dolayısıyla maksimum mükemmellik değerin bir miktarıdır. Rasmussen değerin miktar/derecesinden neyi anladığını örneklendirir: biraz değerli olmak, çok değerli olmak, herhangi bir N derecesinde değerli olmak… 

Rasmussen’in argümanının saldırıya en açık öncülü sonuncusudur. Lehinde öne sürülen alt argümandaki sorun ise ilk öncülünden ileri gelmektedir, maksimum mükemmelliğin muayyen bir miktar değer olup olmamasından. Bu konuda sorulabilecek ilk soru, doğal olarak, neden öyle olduğunu düşünmemiz gerektiği olmalıdır. “Maksimum”; biraz, çok veya herhangi bir N derecesi gibi sıfatlardan önemli bir noktada ayrılır. Örneğin maksimum doğal sayı muayyen bir sayıya tekabül etmez. Dolayısıyla da birbirinden salt nicelik yönünden ayrılan durumların tümünün örneklenebilir olduğunu kabul etmemiz, bu durumların niceliği ifade eden parçasının maksimum değerini aldığı durumun da örneklenebilir olduğunu kabul etmemizi gerektirmez. Bir maksimum noktası olan ve olmayan böyle önermeler/durumlar kümeleri vardır. Öncülün desteklenmesi için de dolayısıyla değer için bir maksimum noktanın söz konusu olduğu gösterilmelidir. Aksi halde argüman en basit ifadeyle eksiktir.

Dikkat çekilebilecek ikinci nokta, değerin maksimum noktasının varlığını göstermenin aksini, böyle bir noktanın olmadığını düşünmek için makul bir gerekçe sunmayı Rasmussen’in ana argümanının diğer öncülleriyle yapabiliriz. N miktarda değer ile N+1 miktarda değer salt nicelik bakımından farklıdır ve bu her N değeri için geçerlidir. Öyleyse (12) gereği her N miktar değer için kendisinden daha fazlası, N+1 miktarda değer de mümkün olmalıdır. Dolayısıyla elimizde değerin maksimum noktası olmadığını düşünmek için bir sebep vardır. (12)’nin dayandığı ilkeden vazgeçilebilir olarak bahsetmiştik. Bunun anlamı ilkenin ifade ettiği iddianın bir nevi varsayılan pozisyon olması ve aksi yönde gerekçeler olması durumunda terk edilebileceğidir. Aksi gösterilmedikçe, bazı miktarlarda değer örneklenebiliyorsa tüm miktarlarda değer örneklenebilmelidir. Aksi gösterilmedikçe, değerin maksimum noktasının olmadığını düşünmeliyiz. Öyleyse Rasmussen’in argümanı için karşılanmamış bir kanıt yükümlülüğünden daha büyük bir probleme sahibiz. 

Son olarak Rasmussen’in argümanına tanrısal sıfatlar hakkına çelişki argümanları üzerinden kurulan itirazı değerlendirmesini inceleyelim. İtiraz, tanrısal sıfatların bireysel veya kendi aralarında tutarsız olduğu iddiasındaki argümanlardan başarılı olanların olması durumunda modal ontolojik argümanın imkan öncülünün hatalı olacağını ifade eder. Rasmussen bir miktar –ancak yalnızca bir miktar- haklılık payıyla bunun kendi argümanının hiçbir ağırlık taşımadığını göstermediğini söyleyerek cevaplar. Elbette ateizm lehine bir argüman doğruysa yalnızca modal ontolojik argüman değil teizm lehine tüm argümanlar yanlıştır ve tersi de geçerlidir. Ancak bu bir argümanı savunmak için karşıt tüm argümanları çürütmek zorunda olduğumuz anlamına gelmez.

Rasmussen bir argümanı savunmak için taşıdığımız yükümlülükler noktasında haklı olsa da burada onun argümanına özgü bir sorunun söz konusu olduğunu söyleyebiliriz. İkinci öncülünü dayandırdığı modal devamlılık ilkesi, kendisinin de ortaya koyduğu üzere nicel yönlerden ayrılan önermeler kümesinin tutarsız üye içermemesini gerektirir. Örneğin kapalı geometrik şekiller bir veya iki kenarlı olamaz, bu tutarsızdır ve kapalı geometrik şekillerde bir ve iki kenarlılık için Rasmussen’in deyimiyle modal boşluk söz konusudur. Bundan dolayı Rasmussen ilkeyi bunun gibi karşı örneklere bağışık olacak şekilde formülize etmiştir ve ilkenin modal devamlılığa işaret etmesinin bir şartı kümenin tutarsız eleman içermemesidir. Ancak sorun tam da tanrısal sıfatlar hakkındaki tutarsızlık argümanlarının değer kümesinde kapalı geometrik şekil örneğinde olduğu gibi modal boşluk olduğu hakkında şüphe uyandırmasıdır. Söz konusu argümanların ne kadar güçlü olduğu, dolayısıyla konuyla ilgisiz değildir ve Rasmussen’in ikinci öncülünün gücünü belirlemektedir.

3. Ateistik İmkan Öncülü Lehine Argümanlar

Ateistik imkan öncülü için inceleyeceğimiz iki argüman da yine ateistik bir argüman geleneği olan kötülük probleminin parçalarını içeriyor olacak. Kötülüğün varlığının Tanrı’yla çeliştiğini ifade eden mantıksal kötülük problemi, dünyada kötülüğün var olduğuna dair öncülü sarsılmaz ölçüde güçlü olsa da teizmin kötülükle uyuşmazlığına dair öncülünde temellendirilmesi zor bir iddia ortaya koymaktadır. Her türlü kötülüğün teizm ile mantıksal açıdan tutarsız olduğunu savunmak güçtür. Öte yandan delilci kötülük problemi uyuşmazlıkla ilgili iddiayı zayıflatır ve Tanrı’nın hiçbir kötülükle değil ancak özel bir tür kötülük olan gereksiz kötülükle bir arada var olamayacağını söyler. Gereksiz kötülük,  Tanrı’nın bu kötülüğe ağır basacak ölçüde büyük bazı iyileri kaybetmeksizin veya en az bu kötülük kadar büyük başka kötülüklere yol açmaksızın engelleyebileceği türden kötülüktür. Teizm ve kötülük arasındaki gerilime dair öncül delilci argümanla birlikte makul bir seviyeye gelmiş olsa da delilci argüman bu ilerlemeye mantıksal argümanda savunulması kolay olan, kötülüğün varlığıyla ilgili öncülü daha zor bir hale getirerek sahip olur. Gereksiz kötülüğün var olduğunu söylemek, basitçe kötülüğün var olduğunu söylemekten çok daha zordur ve argümanın eleştirmenleri de itirazlarını buraya yoğunlaştırmışlardır.

Kötülük problemine modal ontolojik argümanla yaklaşmanın cazip noktası, bir öncüllerini diğer öncülün gücünden taviz vererek kuvvetlendiren bu kötülük argümanlarının tavizlerini daha da azaltmasıdır. İnceleyeceğimiz argümanlar hem uyuşmazlıkla ilgili öncüllerini delilci argümandan alır (veya ona benzer) hem de delilci argümanın aksine gereksiz kötülüğün gerçekten var olduğunu öne sürmez; mantıksal argümandaki kadar olmasa da savunulması bundan daha kolay bir sav ortaya koyarlar: gereksiz kötülük mümkündür.

Şimdiye kadar tartıştığımız argümanlarda önermelerin doğruluklarının mümkün olduğunu göstermek için birçok yol inceledik. Şimdi tartışacağımız argümanların birinde incelenen yöntemlerden biri olan modal devamlılığı kullanacağız. Diğerinde ise şimdiye dek karşımıza çıkmayan farklı bir yöntem kullanacak ve özgür iradeden imkana geçeceğiz.

3.1. Acı Spektrumu Argümanı

Acıyla ilişkili üç özellikle ilgileneceğiz: büyüklük, devam etme süresi, dağılım. Bu üç özellik de niceliksel olarak karşımıza çıkıyor. Örneğin başınızın düne kıyasla daha çok ağrıdığını söylemeniz anlamlıdır. Orman yangınının ortasında kalmış bir hayvanın ölmeden önce diğer bir hayvandan daha uzun süre acı çektiğinin ifade edilmesi de öyle. Son olarak, belli genetik özellikteki hayvanların bir virüsten dolayı acı çekmeye daha yatkın olması da anlamlı bir ifadedir. Acının dağılımı yine niceliksel bir ilişki olan oranla ilgilidir. Bu üç özelliğin farklı niceliksel değerlerini temsil etmesi için kısaca acının derecelerinden bahsedelim.

  • (16) Acının (büyüklük, gerçekleşme süresi ve dağılım bakımından) bazı derecelerinin örneklenmesi mümkündür.
  • (17) Acının bazı derecelerinin örneklenmesi mümkünse tüm derecelerinin örneklenmesi mümkündür.
  • (18) Dolayısıyla acının tüm derecelerinin örneklenmesi mümkündür. (16 ve 17)
  • (19) Acının gereksiz kötülük teşkil eden dereceleri vardır.
  • (20) Öyleyse gereksiz kötülüğün örneklenmesi mümkündür. (18 ve 19)
  • (21) Tanrı ve gereksiz kötülüğün aynı anda örneklenmesi mümkün değildir.
  • (22) Dolayısıyla Tanrı’nın örneklenmemesi mümkündür. (20 ve 21)

Birinci Öncül: Acının bazı derecelerinin örneklenmesi mümkündür.

Acının bazı derecelerinin örneklenebilir olduğunu düşünmek için belki de en iyi neden acının bazı derecelerinin gerçekten örneklemiş olmasıdır. Fiili dünyada acının var olduğu kuşkusuzdur. Herhangi bir miktarda ve süreyle acı hisseden varlığa denk gelmiş olmamız kaçınılmaz. Bu acının belirli bir varlıkta/varlıklarda toplanmış olması da söz konusu. Dolayısıyla, var olanın mümkün olacağına dayanarak söyleyebiliriz ki acının bazı dereceleri örneklenebilirdir.

İkinci Öncül: Acının bazı derecelerinin örneklenmesi mümkünse tüm derecelerinin örneklenmesi mümkündür.

İkinci öncül modal devamlılık ilkesine ve acının tartışılan özelliklerinin niceliksel olmasına dayanır. Modal devamlılık ilkesine göre salt nicelik yönüyle birbirinden ayrılan önermeler imkan bakımından eşdeğer olmalıdır. Örneğin iki nesnenin aynı konumda bulunması ile üç veya daha fazla nesnenin aynı konumda bulunmasının ya birlikte mümkün ya da yine birlikte imkansız olmasını beklemek rasyonel görünüyor. Öte yandan, salt nicelik yönüyle ayrılsa da imkan bakımından tekdüze olmayan önerme sınıfları akla gelebilir. Rasmussen’in argümanını incelerken de değerlendirdiğimiz örneğe göre üç, dört veya daha fazla kenarlı kapalı şekiller mümkünken iki veya bir kenarlı kapalı şekiller mümkün değildir. Ancak bu örnek göstermektedir ki niceliksel farklılık imkan ile bir yerde ilişkili olmalıdır. İki ve bir kenarlılık, kapalı bir şekil olmakla çelişmektedir. Diğer taraftan acının derecelerinde bu tarz, imkan bakımından fark teşkil edecek farklılıklar görünmüyor. 

Modal devamlılık ilkesiyle ilgili önemli bir nokta, onun kapalı şekil örneğindeki nedenle veya farklı nedenlerle vazgeçilebilir olduğudur. İlke bir yerde, aksi yönde bir gerekçe söz konusu olmadığı müddetçe salt nicelik bakımından farklı önerme sınıflarının modal olarak tekdüze olduğunu düşünmek için neden verir. Bu bir nevi varsayılan pozisyondur. Öyleyse söyleyebiliriz ki aksi yönde gerekçemiz olmadığı sürece acının bazı derecelerinin örneklenebilir olmasından her derecesinin örneklenebilir olduğunu çıkarabiliriz.

Üçüncü Öncül: Acının gereksiz kötülük teşkil eden dereceleri vardır.

Acı içsel olarak kötüdür: kötü, istenilmez oluşu başka ek zararları, kötülükleri beraberinde getirebileceği için değil –böyle olsa bile- sırf kendi başına kötüdür. Ancak bir acı; toplamda iyi, tercih edilir bir durum da yaratabilir. Gereksiz kötülük tanımını hatırlayalım. Bir kötülük eşit derecede veya daha büyük bir kötülüğün engellenmesi için ya da büyük bir iyiliğin varlığı için zorunlu değilse gereksizdir. Bu bakımdan üçüncü öncülde ifade edilenin gereksiz kötülüğün acının bir derecesi olduğu değil, acının bazı dereceleri için -açıkça ifade etmek gerekirse bazı miktarlarda, bazı sürelerle gerçekleşen ve bazı oranlarda dağılmış acı için- gereksiz kötülüğün bir emsalinin söz konusu olacağı olduğuna dikkat edilmelidir. Bir diğer deyişle öncülde söylenen şudur: bazı n dereceleri için, <n derecede acı örneklenmiş ise gereksiz kötülük örneklenmiştir> doğrudur. Yani bazı n dereceleri için acının ortaya çıkardığı kötülük, eşit derecede veya daha büyük bir kötülüğün engellenmesi yahut kendisine ağır basacak ölçüde büyük bir iyiliğin varlığı için gerekli değildir. 

Öncül, n’in muayyen bir düzey olduğunu veya n ile temsil edilebilecek düzeylerin kesin bir düzeyin üzerinde olduğunu iddia etmeyi de zorunlu olarak gerektirmez. Acının hangi dereceden itibaren gereksiz kötülüğe örnek oluşturacağı, renklerin sınırlarının bulanıklığına benzer olarak kesin bir şekilde belirlenebilir olmayabilir. Ancak yine renk sınırlarının bulanıklığına rağmen bazı renklerin kesin olarak kırmızı, bazılarının sarı olduğundan bahsedebildiğimiz gibi bazı n dereceleri için de acının gereksiz kötülüğün örneği olduğunu söyleyebiliriz.

Öncülün savunusu konusunda neler söyleyebiliriz? Her şeyden önce, gerçekten bir argümana ihtiyaç duyduğundan kuşkuluyuz. Öncülü apaçık (self-evident) görecek hatrı sayılır sayıda kişi olduğunu düşünüyoruz. Eğer acının bazı derecelerinin gereksiz kötülük teşkil ettiği yanlışsa acının hiçbir derecesinin gereksiz kötülük teşkil etmemesi gerekir ancak bu makul olmaktan son derecede uzak görünüyor. Rowe’un Sue ve Bambi örnekleri gibi gerçekten var olan bazı kötülüklerin, arkalarındaki iyi nedeni bilemiyor olsak da en azından gereksiz göründüğü söylenebilir. Delilci kötülük probleminin vakıa ile ilgili öncülü hakkındaki önemli bir tartışma odağı, kötülüklerin gereksiz görünüyor oluşundan gerçekten gereksiz olduğuna varılıp varılamayacağı ile ilgilidir. Şüpheci teistler bu geçişin problemli olduğunu iddia etmiş ve muhalifleri tarafından ahlaki sezgilerimizin güvenilirliğine dair inancımızla çatışma halinde olmakla eleştirilmişlerdir. Şüpheci teizmi eleştirenlere göre bazı kötülüklerin bariz olarak gereksiz görünmesinin, onların gerçekten gereksiz olduğuna inanmamızı makul kılamaması olayların iyilik ve kötülük içerimlerine dair değerlendirmeler yapmamızı sağlayan ahlaki sezgilerimizin güvenilir olmadığı anlamına gelecektir ki bu makul bir sonuç değildir. Konu hakkındaki tartışma oldukça geniş olsa da şu andaki amacımız açısından bizi ilgilendiren farklı bir sezgimizdir. Acının bazı derecelerinin gereksiz kötülük örneği olacağını ifade ederken bize gereksiz görünen kötülüklerin gerçekten gereksiz olduğundan bahsetmiyoruz. Daha da mühimi, argümanı basitçe gereksiz kötülüğün var olabilirmiş (mümkünmüş) gibi görünmesine de dayandırmıyoruz, karşımızdaki metafiziksel imkana ilişkin anlamıyla modal bir sezgi de değil. Nasıl tanımlanacağı konusunda kuşkulu olsak da sezgilerimiz (i) gereksiz kötülüğün -var olmasının metafiziksel açıdan mümkün olup olmamasından bağımsız olarak- tutarlı bir konsept olduğu (örneklenebilsin veya örneklenemesin, anlamlı, ulaşılabilir bir düzey olduğu) ve (ii) acıyla güçlü bir biçimde ilişkili olduğu yönünde. Evrenin sahip olduğu fiziksel sabitlerin, bir nedenle, aslında metafiziksel anlamda şu an oldukları gibi olmak zorunda olduğu ortaya çıksaydı dahi günümüzdeki hassas ayar tartışmalarına referansla bazı fiziksel sabitler için evrenin yaşam dostu olmayacağını söyleyebilirdik. Benzeri, acının dereceleri ve gereksiz kötülük için söz konusudur.

Geniş çapta kabul gören özgür irade, ruh oluşturma, ceza, doğa yasası gibi teodiselerden hiçbiri acının tüm derecelerini aklayacak, bir anlamda koz kartı gibi her durumda ağır basacak iyiler ortaya koymazlar. Uç örnekleri değerlendirmek bu noktada faydalı olabilir. Yalnızca hissedebilen ancak rasyonel ve özgür irade sahibi olmayan hayvanların yaşadığı bir gezegende (dağılım) bu hayvanlardan birçoğunun (büyüklük) tüm gezegeni etkisi altına almış yangında uzun süreler boyunca (süre) büyük acılar (yeniden büyüklük) çekiyor olduğunu keşfetseydik bunun muhtemelen daha büyük iyiler için gerekli olduğunu düşünmezdik. En önemlisi, böyle bir durumda rasyonel varlıkları gerektiren özgür irade, ruh oluşturma veya ceza gibi teodiseler etkisizdir. Doğa yasası teodisesi ise acının ne denli fazla olacağı noktasında bir sınır olmadığı için yeterli açıklamayı sunamaz. Elbette hayvanların sayısı ve acı çektikleri süre, toplam acıyı arkasında iyi bir gerekçe olabileceğini düşünmemizi olanaklı kılacak bir seviyede olabilir. Ancak açık görünüyor ki olmayabilir de: bu sayı ve sürenin çok fazla olduğu durumlarda bu olanaklılık söz konusu olmayacaktır. Benzer fikir hattı, uzun süreli ve büyük acıları çocukların veya (hatta) rasyonel olanlar da dahil olmak üzere tüm hissedebilen canlıların çektiği durumlar için de kurulabilir. Zira, yeniden, acının derecesinin bir sınırı yoktur ve muhtemelen her türlü acıyı gerekçelendirebilecek bir iyi de yoktur.

İkincisi, şimdiye dek oluşturulmuş teodiselerin her türlü acıyı gerekçelendirebilecek iyiler ortaya koymamasının yanında bu teodiselerin varlığının da başlı başına acının bazı derecelerinin gereksiz kötülük teşkil ettiğine yönelik sezgimizin yaygınlığını gösterdiği söylenebilir. Delilci kötülük problemi üzerine inşa edilmiş devasa bir literatürün varlığı, problemin en azından teoride mümkün olduğunu düşünmek için bir neden veriyor. Eğer her büyüklük, süre ve dağılıma sahip acının daha büyük bir iyilik için gerekli olduğu doğru olsaydı muhtemelen verili acı örneklerinin neden daha büyük iyilikler için gerekli olabileceğine yönelik açıklamalar sunan savunma ve teodiseler oluşturmak gereksiz bir uğraş olurdu. Bahsedildiği gibi savunma ve teodiselerden hiçbiri –belki de neredeyse hiçbiri- acının hiçbir mümkün boyutunun gereksiz olmadığını gösterme amacı gütmüyor. Hedeflenen daha ziyade bahis konusu acıların arkalarında hiçbir iyi gerekçe taşımayan türden acılar olmadıkları. Bu anlamda teist yaşam görüşünde ciddi acıların artlarındaki mümkün veya muhtemel anlamları araştıran projeler olarak savunma ve teodiselerin anlamlı görünmesi, acının bazı derecelerinin gereksiz kötülük teşkil edebileceğine yönelik sezgimizin teist felsefeciler tarafından da paylaşıldığına işaret ediyor.

Öncül lehine bir başka argüman, acının bazı derecelerinin gereksiz kötülük oluşturmasının belirli olaylar karşısında takındığımız tavırların en iyi açıklamasını sunduğuna dikkat çekerek ortaya konabilir. Asistanınızı küçük bir çocuğa aşı yaparken izleyen bir doktor olduğunuzu düşünün. Açıktır ki çocuğun acı hissetmesine karşın asistanınıza müdahale etmiyor oluşunuzun nedeni aşının çocuğun iyiliği için gerekli oluşudur. Bunun argüman bağlamındaki terminolojiyle ifadesi çocuğun tecrübe ettiği acının gereksiz olmamasıdır. Şu noktaya dikkat çekmeliyiz: çocuğun tecrübe ettiği acının asla gereksiz olamayacağını düşündüğünüz için değil, yalnızca acı onun için gereksiz bir kötülük ortaya koyacak düzeyde olmadığı için müdahale etmiyorsunuz. Bu ise acının yoğunluğuyla ilgili bir değerlendirme yaptığınızı ve hissedebileceği acının bazı düzeylerinin çocuğun iyiliği için gereksiz olacağına inandığınızı ima ediyor. Diğer taraftan benzer koşullarda aşı yapılan çocuğun bu kez çok daha fazla acı hissettiğini tahayyül edin. Bu varsayımsal durumda asistanınıza müdahale edip etmeme konusunda tereddüt ediyor oluşunuzun –belki daha uygun ifadeyle tereddüdünüzde haklı oluşunuzun- en iyi açıklaması kararsızlığa konu olan durumun gereksiz kötülük teşkil etmesinin mümkün oluşudur zira acının hiçbir düzeyinin gereksiz, elimine edilmesi gereken türden kötülük teşkil etmesi mümkün olmasaydı endişelenmenizi gerektiren bir durum da olmazdı. Bu kez daha kesin olalım ve Tanrı’nın gereksiz kötülüğü elimine edeceğine veya herhangi başka bir sebebe dayanarak gereksiz kötülüğün metafiziksel anlamda mümkün olmadığına inanan bir itfaiyeci olduğunuzu, orman yangınına müdahale ederken Rowe’un Bambi’si ile karşılaştığınızı ve onu kolayca kurtarabilecek konumda olduğunuzu hayal edelim. Gereksiz kötülüğün imkansız olduğuna inanmanıza karşın Bambi’yi kurtarmanızın arkasındaki en bariz neden, yeniden, acının bazı derecelerinin gereksiz kötülük teşkil edeceğidir. Belki de Tanrı’nın sizi oraya yavru geyiği kurtarmak için gönderdiğini, aynı zamanda bunu yapmasanız dahi Tanrı’nın başka planları olduğunu ve hayvanın başına gelenlerin son tahlilde gereksiz kötülük teşkil etmeyeceğine inanıyorsunuz. Tanrı’nın varlığı ve gereksiz kötülükle ilişkisi hakkında haklıysanız bu inancınızda da haklı olmanız muhtemel ve bu öncülün doğruluğunu yeniden ortaya koyuyor: ormana geyiği kurtararak neyi engellemeye aracılık etmek için gönderildiğinizi düşünüyorsunuz?

Dördüncü Öncül: Tanrı ve gereksiz kötülüğün aynı anda örneklenmesi mümkün değildir.

Öncül delilci kötülük probleminin, tartışmanın iki kutbu tarafınca da geniş oranda kabul edilmiş teleolojik öncülünün bir ifadesinden ibaret. Teleolojik öncülün klasik savunusu kendi başına makul görünen aşağıdaki ilkeye dayandırılabilir:

Bir kimse bir kötülüğün meydana gelmesini engelleyebilecek durumdaysa –o kötülüğe izin verilmesi kötülüğe ağır basacak bir iyilikle sonuçlanmıyor veya kötülüğün engellenmesi en az onun kadar büyük bir kötülüğe yol açmıyorsa- söz konusu kötülüğü engellemelidir.

Ahlaki ilkenin makul oluşu, gereksiz kötülüğün tanımı ve Tanrı’nın kadir-i mutlak ve ahlaken kusursuz doğası birlikte değerlendirildiğinde Tanrı’nın gereksiz kötülüğe engel olacağı söylenebilir. Tanrı bu doğasına zorunlu olarak sahip olduğuna göre çıkarılan sonuç da zorunludur. Tanrı’nın varlığı gereksiz kötülüğün var olmamasını gerektirir. Bu yüzden (21) doğrudur.

Özetle argümanın onu değerlendiren kimse açısından işleyişi şu şekildedir: değerlendirmeci bazı büyüklük, süre ve dağılımlarda acıya şahit olmuştur; büyüklük, süre ve dağılımdaki gibi nicel farkların modal statü bakımından önemsiz olduğuna dair ilke dolayısıyla çok daha yoğun, uzun süren ve orantısız dağılmış acının söz konusu olması da mümkündür, acı spektrumundan herhangi bir nokta örneklenebilir; ancak bu çok daha yoğun, uzun ve orantısız dağılmış acılar arasında artlarında daha büyük iyiler taşımayanlar da vardır, acı spektrumundaki her nokta adil ve kabul edilebilir değildir. Örneğin değerlendirmeci belirli bir kötülük için bir teodiseyi makul bulur ve onun gereksiz olmadığına ikna olur. Ancak niceliğin imkanda belirleyici olmadığını görerek kötülüğün –fiilen olmasa da- gereksiz olabileceğini fark eder. Gereksiz kötülük teizm ile bağdaşmaz. Ateizm lehine modal ontolojik argümanın öncülü için gerekçelendirme tamamlanmıştır.

İtirazlar

İlk itiraza göre ikinci öncülde kullanılan modal devamlılık ilkesi mutlak bir ilke değildir, salt nicelik bakımından ayrılan önerme sınıflarında modal boşluklar olabilir –yani bazı önermelerin modal statülerinde farklılık olabilir. Dolayısıyla modal devamlılığı kırmak için bir gerekçemiz olduğu ölçüde öncülü reddetmek için bir gerekçemiz de vardır. Tanrı varsa basitçe acının örneklenemeyen dereceleri olacaktır, bunlar gereksiz kötülük teşkil edenlerdir. Dolayısıyla Tanrı’nın varlığını kabul etmek için gerekçelerimiz, ikinci öncülü reddetmek için de neden teşkil eder.

Gerçekten de Tanrı varsa, (21) gereği, acının bazı derecelerinin örneklenmiş olmasından tüm derecelerinin örneklenebileceği sonucuna ulaşamayız. Öte yandan itirazın haklı olması güçlü olduğu anlamına gelmez. Elbette Tanrı varsa, ateistik modal ontolojik argümana bağlanacak argümanın öncüllerinden en az biri hatalı olmalıdır. Dahası, Tanrı varsa, “Tanrı yoktur.” ile sonuçlanan tüm argümanlar hatalı olacaktır. Tabii ki tersi de geçerlidir. Yine de bunların hiçbiri argümanın kendi başına sonucu lehine pozitif bir destek sunduğu gerçeğini değiştirmez. Bir argümanın güçlü olduğunu söyleyebilmek için karşı görüşün tüm argümanlarını çürütmek zorunda değiliz. Argümanın yaptığı katkı, teistik argümanların güçlü olmadığı gösterilmeden de kabul edilebilir. Aradığımız, teist ve ateist kampların imkan öncülleri arasındaki pat durumunu çözmeye yardımcı olacak bir argüman bulmak olduğuna göre acı spektrumu argümanının bu gerekliliği yerine getirdiğini ve dolayısıyla başarılı olduğunu söylemeliyiz. Elbette teistik argümanlar güçlü değilse ikinci öncülün, dolayısıyla da argümanın zemini daha da sağlam hale gelebilir ancak bu, şu aşamada zorunlu değildir.

Yukarıdaki gibi mütevazı bir cevapla yetinmeyip argümanın taşıdığı ağırlığı sırf kendi başına değil ancak diğer argümanlar da hesaba katıldığında argümanın ne gibi bir yerde olduğunu da tartışabiliriz. Modal devamlılığın kırıldığını düşünmek için sahip olabileceğimiz gerekçe Tanrı’nın var olması idi. Buradaki tartışmayı bir adım yakından inceleyelim. Bir tarafta bir P önermesinin –gereksiz kötülüğün varlığı- mümkün olduğunu ifade eden argüman, diğer tarafta başka bir Q önermesinin –Tanrı’nın varlığı- zorunlu olarak doğru olduğunu ifade eden bir pozisyon varken P ile Q’nun bir arada doğru olamayacağını biliyoruz (21). İlk olarak sezgilere bakabiliriz. Bize ◊P (mümkündür ki gereksiz kötülük vardır) önermesi, ◊Q (mümkündür ki Tanrı vardır) önermesinden çok daha sezgisel görünüyor. Bir teist ile de burada hemfikir olmamız gerektiğinin en temel sebebi, Tanrı’nın (daha net olmak gerekirse maksimum mükemmelliğin) tanımı gereği, ◊Q’nun □Q’ya (zorunlu olarak Tanrı vardır) denk olmasıdır. Ya zorunlu olarak doğru ya da zorunlu olarak yanlış olabilecek bir önermenin sezgiselliğinin önermenin imkan dahilinde oluşu fikrine verdiği destek olumsal bir önermeninkinden çok daha zayıftır. Örneğin Fermat’ın Son Teoremi’nin sezgisel olarak doğru olabilir gibi görünmesi doğru olduğuna inanmak için olumsal bir önermede olduğu kadar güçlü bir sebep vermezdi. Burada ◊~Q’nun da en az ◊Q kadar sezgisel olduğuna dikkat çekilebilir. P (mümkünse) olumsal olacağı için hem ◊P hem de ◊~P doğru olabilirken ◊Q ile ◊~Q arasından yalnızca biri doğrudur. Dolayısıyla Tanrı’nın varlığının mümkün veya zorunlu olduğunun kendisine sezgisel olarak doğru göründüğünü ifade ederek gereksiz kötülüğün imkanını reddetme yoluna başvuracak birinin yöntemi makul değildir. İkincisi, ◊P ile ◊Q’den birinin yanlış olduğunu kabul edeceksek bunun ◊Q olduğunu düşünmek için nedenlerimiz olduğu da söylenebilir. Tanrı’nın içsel tutarlılığı din felsefesinde başlı başına büyük bir tartışma alanıyken gereksiz kötülüğün içsel olarak tutarsız olduğuna dair bir iddia yoktur, sezgilerimiz gereksiz kötülüğün içsel olarak tutarlı olduğunu güçlü bir biçimde desteklemektedir.

Üstelik ◊P’nin sezgisel olarak doğru görünmesi bir yana, argümanda bu önermenin direkt olarak bu önermeye yönelik sezgilerle temellendirilmediğine dikkat edilmelidir. Argüman ◊P’ye ondan daha sezgisel üç öncülle ulaşıyor. Bu öncüllerin her birinin, yukarıdaki tartışma da göz önünde bulundurulduğunda, ◊Q’den çok daha sezgisel olduğunu düşünüyoruz. Acının dereceleri için modal devamlılığın söz konusu olması, bizce maksimum mükemmellikte bir varlığın mümkün veya zorunlu olmasından çok daha makuldür.

Son olarak, ◊Q lehine argümanlar sunulabilir. Doğrudan bu önermeye yönelik argümanlardan birçoğunu önceki bölümlerde geniş biçimde tartıştık ve hiçbirinin makul olmadığını savunduk. Ancak farklı olarak Tanrı’nın varlığı lehine argümanlar da tartışmaya dahil edilebilir. Buradaki sorun, teist ve ateist argümanlar arasında konu açısından bir asimetri olmasıdır. Tartışmadaki modal devamlılığı kırmak için teist argümanların zorunlu, maksimum mükemmellikte bir varlıkla sonuçlanması gerekir ancak birçoğu maksimum mükemmelliğe yakın ancak zorunlu olmayan bir varlıkla uyumludur. Öte yandan ateist argümanlar maksimum mükemmelliğe yakın bir varlığın gerçekte var olmadığı sonucuna ulaşarak doğal olarak maksimum mükemmellikte zorunlu bir varlığın olmadığı sonucuna ulaşır. Bundan dolayı da –tartışmayı çok daha kompleks hale getirecek olsa da- diğer teist ve ateist argümanları işin içine dahil etmek, ◊Q önermesini savunan ve modal devamlılığa itiraz eden taraf için iyi bir yol değildir.

İkinci bir itiraz teleolojik öncüle getirilebilir. Peter van Inwagen, bu öncülün dayandırıldığı ahlaki ilkeye karşı bir akıl yürütme getirmiştir11. Suçluya on yıl hapis cezası vermek üzere olan bir hakim hayal edelim. On yıllık bir ceza ile on yıl eksi bir günlük bir ceza arasında cezanın amaçları açısından (caydırıcılık veya tatmin gibi) bir fark yoktur. Öyleyse ahlaki ilkeye göre hakimin bir gün eksik olan cezayı vermesini bekleriz. Ancak sorun odur ki bu değerlendirme bir günle sınırlı değildir; aynı şekilde on yıl eksi iki gün için de düşünülebilir. Bu yol takip edildiğinde ise nihayetinde suçluya ceza verilmemesi ile karşılaşırız fakat açıktır ki ceza verilmelidir. Öyleyse keyfi de olsa bir yerde sınır çizilmek zorundadır. Bu durum, hiçbir cezanın adil olmadığı sonucunu doğurduğu için ilkenin hatalı olduğu anlamına gelir.

Kötülük problemine dönmek gerekirse, van Inwagen Tanrı’nın amaçları açısından dünyadaki kötülüklerin tümünü elimine edemeyeceğini, bir miktarına izin vermesi gerektiğini göstermenin ardından benzer durumun burada da söz konusu olduğuna işaret etmiştir. Bir miktar kötülüğe izin vermesi gereken Tanrı da keyfi bir çizgi çekmelidir. Bu da dünyadaki kötülüklerden bazılarının tanım itibariyle gereksiz olacağı anlamına gelir.

van Inwagen’in müstakil bir yazıda tartışılabilecek bu argümanına kendi amacımız açısından daha basit bir karşılık verebiliriz. Hakim örneğinde hiçbir düzeydeki cezanın adil olmadığı sonucunun hatalı olması gibi her düzeydeki cezanın adil olduğunun da yanlış olduğunu söylemeliyiz. Sınır çekme problemiyle karşı karşıya olduğumuzu kabul etsek bile bu on yıllık olması düşünülen ceza yerine müebbet hapis cezası vermenin haksız olacağını hala söyleyebiliriz. Paralel olarak Tanrı’nın izin vereceği kötülük miktarında bir keyfilik olabilmesini onaylasak dahi bu çok fazla kötülüğe izin verebileceği anlamına gelmez. Bu tarz bir cevap, delilci kötülük probleminin kanıt yükünü artırabilecek olsa da (yani bundan böyle dünyada gereksiz kötülüğün var olduğunu değil çok fazla gereksiz kötülüğün olduğunu göstermeyi gerekli kılsa da) tartıştığımız gerçekten var olan kötülükler değil ancak var olması mümkün olan kötülükler olduğu ve ilk iki öncülle birlikte acının tüm düzeylerinin örneklenebilir olduğu sonucuna ulaştığımız için argümanımız açısından ek bir kanıt yükü altına giriyor değiliz. van Inwagen’in fikir hattını kabul etsek dahi hala (a) Tanrı ile gereksiz kötülüğün bazı miktarları bir arada bulunamaz ve (b) gereksiz kötülüğün Tanrı ile bağdaşmayan bu bazı miktarları örneklenebilirdir. Nihai olarak, güçlü olup olmamasından bağımsız olarak, van Inwagen’in argümanı ateistik imkan öncülünü destekleyen argüman için zararsızdır.

3.2. Ahlaki Sapkınlık Argümanı

Mantıksal kötülük problemine çözüm olarak sunduğu özgür irade savunmasında Plantinga özgür irade sahibi varlıkların bulunduğu ve ahlaki kötülükten çok ahlaki iyilik yaptıkları bir dünyanın özgür irade sahibi varlıklar içermeyen bir dünyadan daha iyi olabileceğini; ancak özgür irade sahibi varlıkların yaratılmasının bir sonucunun ne yazık ki ahlaki kötülük olacağını ifade etmiştir. Bu durumda Tanrı’nın özgür iradeli varlıklar içeren ve çoğunlukla ahlaki iyilik yapılan bir dünya yaratmayı seçmesi mümkündür. Bu anlamıyla da Tanrı ile ahlaki kötülüğün bir arada bulunması mümkün olduğundan mantıksal kötülük problemi teizm için bir tehdit değildir.12

Richard Gale, Plantinga’nın modal ontolojik argümanının imkan öncülüne yine onun genel hatlarıyla bu şekilde ifade edilebilecek özgür irade savunmasının üzerine inşa edildiği öncüllerine dayanan bir itiraz getirmiştir.13 Gale’ın akıl yürütmesi yalnızca bir itiraz boyutunda kalmayıp ateistik imkan öncülü lehine pozitif bir argüman formunda da savunulabilir. Bunu yapmak için önce Plantinga’nın özgür irade savunmasının ardındaki mantık örgüsünü inceleyerek argüman için gerekli kavramlara değineceğiz.

Yukarıda kısaca ortaya konan savunma ile Mackie gibi mantıksal kötülük problemi savunucuları ve Leibniz gibi Tanrı’nın mümkün dünyaların en iyisini fiili kıldığını düşünen teist filozoflar arasındaki temel gerilim, onlara göre Tanrı’nın kadir-i mutlaklığı gereği dilediği mümkün dünyayı fiili kılabileceğidir. Plantinga bu fikri sorgular ve reddeder. Başlamadan önce birkaç detaya dikkat çekilebilir. Tanrı yeryüzünü, cenneti, Sokrates’i yaratırken bunların bulunduğu durumları, dünyaları fiili kılar. Varlıklar yaratılır, dünyalar ise fiili kılınır. İlk bakışta Tanrı’nın fiili kılamayacağı birçok mümkün dünya vardır. Tanrı sizin hiç doğmamış olduğunuz bir dünyayı fiili kılamaz zira bunun için artık çok geçtir. Eğer olumsal bir varlık ise kendisinin bulunmadığı mümkün dünyaları da fiili kılamaz. Ancak bunlar konumuz açısından kritik öneme sahip değildir. Mackie veya Leibniz’in iddia edeceği şey şu olacaktır: Tanrı sizin var olmadığınız bir dünyayı fiili kılabilirdi (could have actualized). Aynı şekilde Tanrı olumsal bir varlık olsa dahi soru, içerisinde var olduğu tüm mümkün dünyaları fiili kılıp kılamayacağıyla ilgilidir.

Plantinga’yı takip ederek Boston Belediye Başkanı Curley Smith örneğini değerlendirelim. Smith bazı yapıların yıkımını gerektireceği için yapılması planlanan otobanın rotasına karşı çıkmaktadır. Çekincesini anlamsız bulan Karayolu Departmanı, itirazını sonlandırması için Smith’e $35,000 rüşvet teklif eder ve Smith ikna olur. Ancak rüşveti teklif eden Karayolu Departmanı Başkanı, Smith’in $20,000’e de tav olup olmayacağını merak ederek uykusuz bir gece geçirir. Sonuç olarak merak ettiğimiz, aşağıdaki önermelerden hangisinin doğru olduğudur:

  • (24) Eğer $20,000 teklif edilmiş olsaydı Smith rüşveti kabul edecekti.
  • (25) Eğer $20,000 teklif edilmiş olsaydı Smith rüşveti kabul etmeyecekti.

Koşullu önermelerin öncül (antecedent) (Smith’e $20,000 teklif edilmesi) kısımlarını genişletip mümkün dünyalar boyutuna geçelim. W ve W*, Smith’in teklife vereceği cevaba (buna t anı diyelim) kadarki bölümleri ortak olan (bu bölüme de S’ diyelim) ancak t anında Smith’in yaptığı özgür iradeli seçimle dallanan iki dünya olsun. Diyelim ki Smith W’de rüşveti kabul ediyor, W*de ise etmiyor. Bu iki dünyanın ortak olarak sahip olduğu S’ parçası (i) Smith’e $20,000 teklif edilmesi de dahil olmak üzere t anına kadarki tüm tarihi ve (ii) Smith’in rüşveti kabul edip etmeme eyleminde özgür olmasını içerir ancak (iii) Smith’in rüşveti kabul etmesini veya etmemesini içermez.

  • (25) S’ geçerlidir.

önermesi

  • (26) Smith rüşveti kabul eder.
  • (27) Smith rüşveti kabul etmez.

önermelerinden ikisini de gerektirmez. Dolayısıyla hem (25) ve (26)’nın doğru olduğu dünyalar (W) hem de (25) ve (27)’nin doğru olduğu dünyalar (W*) vardır. Burada dikkat etmemiz gereken en önemli husus Smith’in t anındaki eyleminin liberteryen anlamda özgür olmasıdır. Smith rüşveti kabul edip etmemekte liberteryen anlamda özgürdür, bundan dolayı hem W hem de W* mümkün dünyalardır. Ancak Plantinga (23) ve (24)’ten birinin ve yalnızca birinin doğru olduğuna dikkat çekerek her halükarda Tanrı’nın fiili kılamayacağı mümkün bir dünya olduğu sonucuna varır. Diyelim ki (23) doğrudur. Bu durumda Tanrı W*yi fiili kılamaz zira bunu yapması için öncelikle W*nin t anına kadarki bölümünü temsil eden S’ parçasını fiili kılması gerekir ancak S’ fiili kılındığında (23)’ün doğruluğu gereği Smith rüşveti kabul eder ve W* fiili kılınmaz. Tanrı’nın Smith’in t anında rüşveti reddetmesine neden olması durumunda ise, S’ Smith’in t’deki seçiminde özgür olmasını içerdiği ve Tanrı Smith’in seçimini aktif müdahale ile özgür olmaktan çıkardığı için S’ fiili kılınmış olmaz, dolayısıyla W* de fiili kılınmış olmaz. Her durumda, mümkün bir dünya olan W*yi fiili kılmak Tanrı’nın kudretinin ötesindedir zira bu mantıksal olarak imkansızdır. Aynıları (23) yerine (24)’ün doğru olması durumunda W için doğrudur. Plantinga’nın bahsi ileride kısaca geçecek ara bir basamakla buradan ulaşacağı sonuç, tüm özgür iradeli varlıkların daima ahlaken iyi olanı yaptığı mümkün dünyalar olsa dahi Tanrı’nın bunları fiili kılamayacağıdır.

Dikkate değer ikinci nokta Tanrı’nın mümkün dünyaları nasıl fiili kıldığı ile ilgilidir. S’ parçasının fiili kılınmasında olduğu gibi dünyanın olumsal parçalarına doğrudan sebep olmaya güçlü fiili kılma (strong actualization), S’ fiili kılınarak Smith’in rüşveti kabul etmesine neden olunması gibi dünyanın olumsal parçalarına dolaylı olarak sebep olmaya da zayıf fiili kılma (weak actualization) diyebiliriz. Zayıf fiili kılma, üzerinden geçmek gerekirse, takip eden süreçle temsil edilebilir. Diyelim ki aşağıdaki önerme doğrudur ve Tanrı’nın bilgisi kapsamındadır. S de S’ gibi söz konusu eylemin gerçekleşeceği ana kadarki maksimal dünya parçası (maximal world segment) olsun:

  • (28) S geçerli ise x özgür biçimde A eylemini gerçekleştirir.

Bu durumda Tanrı x’in özgür biçimde A eylemini gerçekleştirmesini S parçasını fiili kılarak (güçlü biçimde fiili kılarak) sağlayabilir. D, S parçasına sahip olan ve x’in özgür biçimde A’yı gerçekleştirdiği bir dünya ise Tanrı D dünyasını zayıf biçimde fiili kılar.

Bu aşamada Plantinga’nın Tanrı’nın ne yapamayacağı üzerine yürüttüğü tartışmadan farklılaşarak Tanrı’nın ne yapabileceği üzerinde durabiliriz. Her ne kadar (28)’in doğru olması durumunda Tanrı yine S parçasına sahip olan ve x’in A’yı yapmaktan kaçınmasıyla D dünyasından ayrılan D* mümkün dünyasını fiili kılamasa da D’nin fiili olmamasını sağlamak tamamen onun elindedir. Bunun için tek yapması gereken S’yi fiili kılmamaktır –veya D’de içerilmeyen başka bir durumu fiili kılmak. Somutlamak adına, (28) doğruysa ancak Tanrı x’in A’yı gerçekleştirmesini istemiyorsa, söz gelimi x bir münafık ve A da peygamberin yerini onu öldürmeye gelenlere söyleme eylemi ise, Tanrı S’yi fiili kılmaz ve böylece münafık peygamberin yerini bildiremez. Bir ara sonuç olarak, öyleyse, söyleyebiliriz ki yaratılan varlıkların belirli liberteryen özgür iradeli seçimler yapmalarını sağlamak (zayıf olarak sağlamak) her zaman olmasa da belirli seçimleri yapmalarına engel olmak Tanrı’nın kudreti dahilindedir.

Tanrı’nın kudretinin neyi kapsadığına dair çıkarımımızı argümanın ileri aşamalarında kullanacağız. Bundan önce bahse konu özgür iradenin liberteryen olduğunu ve bundan dolayı (28) gibi önermelerin olumsal olduğunu hatırlayalım. Diğer bir ifadeyle (28) doğru olabilir, ancak x’in A eylemi liberteryen anlamda özgür olduğu için zorunlu olarak doğru değildir. Aşağıdaki önerme de mümkün olmalıdır:

  • (29) S geçerli ise x özgür biçimde A eylemini gerçekleştirmekten kaçınır.

Bu noktada yine Plantinga’dan gelen bir tanımdan daha bahsetmeliyiz. A gibi eylemler ahlaken önemsiz olabilir. Örneğin Smith’in rüşvet teklifine cevabı aksine kahvaltıda omlet veya haşlanmış yumurta tercih etmesi ahlaken önemsizdir. Rüşvet, peygamberin yerini suikastçılara bildirme, Tanrı’nın buyruğuna uyma gibi konulardaki eylemler ise ahlaken önemlidir (morally significant). Ahlaken önemli durumlarda özgür irade sahibi varlıklar ise önemli ölçüde özgürdür (significantly free). Son olarak, ahlaken önemli bir tercih/eylem ya ahlaken doğrudur ya da ahlaken yanlıştır. Tanımlar ve imaları ışığında argümanın ilk aşaması aşağıdaki gibidir:

  • (30) Önemli ölçüde özgür, yaratılmış varlıkların bulunduğu mümkün bir dünya vardır.

Özgür irade savunmacıları ve liberteryen özgür iradenin imkanını kabul eden diğer herkesin doğruluğunu teslim edeceği (30)’un bir sonucu ise

  • (31) Öyleyse, önemli ölçüde özgür, yaratılmış tüm varlıkların daima ahlaken yanlış olanı gerçekleştirdiği mümkün bir dünya vardır.

(30)’dan, ateistik imkan öncülü lehine argümanın ilk öncülünü oluşturan (31)’in çıkarılması bir miktar yer ve zamanı hak ediyor. Kelimeleri ekonomik kullanmak adına, önemli ölçüde özgür, yaratılmış tüm varlıkların daima ahlaken yanlış olanı gerçekleştirdiği durumlara “Sapkınlık” adını verelim. Öyleyse amacımız, (30)’dan Sapkınlık’ın örneklendiği mümkün dünyaların olduğuna ulaşmak olacak.

İki gerekçelendirme yapacağız. İlki için özgür irade savunmasına dönmeliyiz. Daha önce Plantinga’nın fiili kılınmaları Tanrı’nın kudreti dahilinde olmayan mümkün dünyalar olduğuna dair argümanını incelemiş ve buna bir basamak ekleyerek tamamen ahlaki iyilik içeren (ahlaki kötülükten azade) bir dünyanın yine Tanrı tarafından fiili kılınamayacağı sonucuna ulaştığından bahsetmiştik. Bu ara basamak Plantinga’nın “transworld depravity” dediği öncüldür. Buna göre mümkündür ki yaratılabilir bazı özgür kişiler için bu kişiler en az bir kez ahlaken önemli bir seçimde ahlaken yanlış olanı yapar. Bunun bazı kişiler için değil, tüm kişiler için doğru olması da mümkündür. Sonuç olarak Plantinga’nın terminolojisiyle, mümkündür ki yaratılabilir her kişi en az bir kez ahlaken önemli bir seçimde ahlaken yanlış olanı yapma niteliğine sahiptir.

Plantinga’nın bu öncülünün makul bulunması durumunda takip eden önermenin de makul bulunması beklenebilir: (a) mümkündür ki yaratılabilir bazı özgür kişiler (en az bir kez değil ancak) ahlaken önemli tüm seçimlerde ahlaken yanlış olanı yapma niteliğine sahiptir. Plantinga’nın öncülü (a)’yı gerektirmez ancak, ifade edildiği gibi, öncülü kabul edip (a)’yı reddetmek makul görünmemektedir. Şüphesiz, bir arada var olabilecek (compossible) yaratılabilir, özgür kişiler vardır. Şu da makul görünmektedir: (b) mümkündür ki dünyadaki tüm özgür kişiler (en az bir kez değil ancak) ahlaken önemli tüm seçimlerde ahlaken yanlış olanı yapma niteliğine sahiptir. Dolayısıyla Sapkınlık’ın örneklendiği mümkün bir dünyanın olduğunu düşünmek, muhtemelen en az “transworld depravity” öncülünü kabul etmek oranında makuldür.

İkinci rota için basitten başlamak adına, önemli ölçüde özgür, yaratılmış tek bir P kişisini (örneğin Adem’i) içeren ve bu kişinin karşı karşıya olduğu, sonuçları birbirinden bağımsız, ahlaken önemli, ardışık üç seçimin olduğu bir dünya düşünelim. P’nin seçimleri haricinde değişmeyeceğini varsayabileceğimiz bu dünya, ilk seçimle birlikte ilk kez dallanacak ve elimizde bundan böyle P’nin ilk seçimine kadarki tarihleri ortak olan iki dünya bulunacaktır. Bunu ikinci seçim izler ve her iki dünya da yine birer kez dallanarak dört dünyadan söz edilebilecek bir tablo ortaya koyar. P’nin ilk seçiminin ahlaken doğru olduğu dünyadan dallanan ikincinin de doğru olduğu ve ikincinin yanlış olduğu iki dünya; ilkinin yanlış olduğu dünyadan dallanan ikincinin doğru olduğu ve ikincinin de yanlış olduğu iki dünya. Bunu son durumdaki dört dünyanın her biri için son bir dallanma izler ve elimizde nihai olarak sekiz dünya bulunur. Bu dünyalardan bazılarında P bazen iyiyi bazen kötüyü izlemişken birinde daima iyiyi, birinde ise daima kötüyü gerçekleştirmiştir. Şimdi, buraya kadarki örneklerde olduğu gibi, P’nin her seçimi için o seçimin ne olacağına dair birer doğru olacaktır. Söz gelimi (ilk seçimi x1 olarak adlandırıp C durumunda -veya maksimum dünya parçasında-, t1 anında gerçekleştirildiğini varsayarsak):

  • (32) C durumu geçerli olursa, P, t1 anında x1 seçiminde ahlaken doğru olanı gerçekleştirir.

Önermesi doğru olsun. Seçimlerde izlenen farklı yolların, devam eden durumları (maksimum dünya parçalarını) da farklılaştıracağı için (C gibi durumların dünyanın seçimin yapıldığı ana kadarki tarihini içerdiğini hatırlayalım) devam eden bu durumları ifade etmede bir yöntem oluşturabiliriz. x1’de ahlaken doğru olanın gerçekleştirildiği yeni durumu C(x1+), yanlış olanın gerçekleştirildiğini ise C(x1-) olarak adlandıralım. Buna göre takip eden üçüncü nesil durumlar da C(x1+)(x2+), C(x1+)(x2-), C(x1-)(x2+) ve C(x1-)(x2-) olacaktır. Sembollerden konuya dönmek gerekirse (32)’nin doğruluğu, C(x1-) durumunu içeren dünyaların fiili olamayacağı anlamına gelir. (32) gibi aşağıdaki önermeler de doğru ise

  • (33) C(x1+) durumu geçerli olursa, P, t2 anında x2 seçiminde ahlaken yanlış olanı gerçekleştirir.
  • (34) C(x1+)(x2-) durumu geçerli olursa, P, t3 anında x3 seçiminde ahlaken doğru olanı gerçekleştirir.

fiili olabilecek tek dünya P’nin sırasıyla doğru, yanlış ve doğruyu gerçekleştirdiği dünyadır. Son olarak bu dünya @, P’nin üç eyleminin de ahlaken yanlış olduğu dünya ise # olarak adlandırılsın. (31)’i temellendirme amacımız gereği # gibi bir dünyanın fiili kılınabilmesi zorunlu değildirdir. İncelemekte olduğumuz örnekte her ne kadar P’nin yapmış olacak olduğu (would have done) seçimler gereği # asla fiili kılınamayacak olsa da P’nin x1, x2 ve x3 seçimlerinde liberteryen anlamda özgür olmasının bir gereği olarak # mümkün bir dünya olmalıdır. (32), (33) ve (34) ancak olumsal doğrular olabilirler.

Bu basit örnekte doğru olan, daha fazla kişi ve daha fazla seçimin dahil edildiği durumlarda da geçerlidir. Değişen yalnızca dünyaların farklılaşma, dallanma sayılarıdır. Söz gelimi yine tek bir kişinin üç yerine yüz seçimle karşı karşıya olduğu bir örnekte birinde tüm seçimlerin ahlaken yanlış olduğu ve yine birinin fiili kılınabileceği 2100 mümkün dünyadan söz edilebilirken aynı durumda iki kişinin olması örneğinde 2101 mümkün dünya vardır. Farklı kişilerin seçimlerinin birbirini etkilediği ve kişilerin seçimleri yapacağı, fiili kılınabilecek farklı C durumlarının olduğu, kısacası gerçeklikteki gibi örneklerde hesap daha karmaşık olacak olsa da sonuç yine geçerlidir: tüm seçimlerin ahlaken yanlış olduğu mümkün dünyalar vardır.

Argümanın ikinci aşaması ise sonucun iması ile ilgilidir.

  • (35) Hem Tanrı’nın var olduğu hem de önemli ölçüde özgür, yaratılmış tüm varlıkların daima ahlaken yanlış olanı gerçekleştirdiği mümkün bir dünya yoktur.

(31) ile birlikte yukarıdaki önermeden ulaşabiliriz ki

  • (36) Tanrı’nın var olmadığı mümkün bir dünya vardır.

(35) nasıl temellendirilebilir? İddiası bir yerde Tanrı ve Sapkınlık’ın bir arada örneklenemez yani tutarsız oluşu olduğuna göre <Tanrı vardır> ve <Sapkınlık vardır> önermelerinin tutarsız olduklarını ortaya koyan ek önermelere ihtiyaç duyuyoruz. Bu önermeler ise şunlardır:

  • (37) Zorunlu olarak, Tanrı kudreti dahilinde olması durumunda önemli ölçüde özgür, yaratılmış tüm varlıkların daima ahlaken yanlış olanı gerçekleştirdiği bir durumun gerçekleşmesini engeller.
  • (38) Önemli ölçüde özgür, yaratılmış tüm varlıkların daima ahlaken yanlış olanı gerçekleştirdiği bir durumun gerçekleşmesini engellemek Tanrı’nın kudreti dahilindedir.

(37), yeniden, kötülük argümanlarının teleolojik öncülleri ile paraleldir. Öyle görünüyor ki şu iki önerme ve onlardan çıkan sonuç makuldür: (p1) Tanrı, kudreti dahilindeyse, biri diğerinden önemli ölçüde daha kötü iki durumu var kılmakla karşı karşıya kaldığında önemli ölçüde daha kötü olan durumu var kılmaz; (p2) Sapkınlık, hiçbir özgür kişi içermeyen bir durumdan önemli ölçüde daha kötüdür; (C) Tanrı, kudreti dahilindeyse, Sapkınlık’ı var kılmaz. Dolayısıyla (37) de makuldür.

Aynı önerme özgür irade savunmasının temel prensiplerinden biri ile ortak sezgilere dayanmasıyla da desteklenebilir. Plantinga’ya göre özgür irade sahibi varlıkların bulunduğu ve ahlaki kötülükten çok ahlaki iyilik yaptıkları bir dünya özgür irade sahibi varlıklar içermeyen bir dünyadan daha iyidir. İtalik ifade, (37) ile aynı doğrultudadır.

(38)’in ilk ayağı, daha önce gösterdiğimiz nokta olan Tanrı’nın her mümkün dünyayı fiili kılmaya kadir olmasa dahi dilediklerini fiili kılmamaya kadir oluşudur. Tanrı, güçlü olarak fiili kılması durumunda Sapkınlık’ın fiili kılınacağı olumsal ve özgür iradeli eylemleri içermeyen bir C durumunu –bir diğer ifadeyle tüm insanların  daima ahlaken yanlış olanı yapacağı durumların toplamını- fiili kılmaktan kaçınarak Sapkınlık’ı fiili olmaktan alıkoyabilir. İkinci ayak ise Tanrı’nın C’yi fiili kılması durumunda Sapkınlık’ın örnekleneceğini biliyor oluşudur. Sonuç olarak Tanrı salt ahlaken yanlış eylemleri içeren bir durumun nasıl fiili olabileceğini ve olmayacağını bilir ve buna göre böyle bir durumun fiili olmamasını sağlayabilir. Yani Sapkınlık’ı engellemek Tanrı’nın kudreti kapsamındadır.

İtirazlara geçmeden önce birkaç noktaya daha değinelim. Bazı nedenlerle (31)’i savunmanın güç olduğu öne sürülebilir. Ancak (31)’in ifade ettiği kadar güçlü bir iddia argüman için zorunlu da değildir. Öncülün (30)’dan benzer bir argümantasyonla çıkarılabilecek zayıf bir versiyonu da aynı işlevi görecektir:

  • (31*) Önemli ölçüde özgür, yaratılmış tüm varlıkların büyük oranda ahlaken yanlış olanı gerçekleştirdiği mümkün bir dünya vardır.

Argümanın diğer öncülleri de daima yerine büyük oranda operatörünü kullanan (31*)’e adapte edilebilir. (31)’deki durumu ifade etmek için kullandığımız Sapkınlık ifadesini de (31*) için “yakın-Sapkınlık” olarak güncelleyebiliriz.

Son olarak, argümanı farklı bir ifadeyle bir bütün halinde şöyle ortaya koyabiliriz: W dünyası; içerisindeki Tanrı olmayan varlıkların liberteryen özgür iradeli eylemlerini içermeyen ve bu varlıkların liberteryen özgür iradeli eylemlerinin içerisinde gerçekleştiği (yukarıdaki C gibi) durumları içeren bir K parçası ve bu varlıkların liberteryen özgür iradeli eylemlerinden oluşan J parçasından oluşuyor olsun. Bir diğer deyişle bu dünyada (Tanrı’nın fiili kılmasıyla veya diyelim ki şans eseri) K fiili olursa J’nin de olacağı doğrudur. Argümana göre J’nin Sapkınlık veya yakın-Sapkınlık’ın bir örneği olduğu mümkün W dünyaları vardır. Yine argümana göre, Tanrı böyle bir dünyada J’nin ortaya çıkmasına izin vermez, dolayısıyla W’nin bir parçası olan K’nin içerisinde Tanrı yoktur. Dolayısıyla Tanrı’nın var olmadığı mümkün bir dünya vardır.

İtirazlar

Argümanın diyalektik gücü sorgulanabilir. Söylenebilir ki argümanın gösterdiği tek şey aslında yalnızca insanların bazı durumlarda özgür olmadıkları veya olmayacaklarıdır (gelecek zaman kipi değil, var olmayan koşullu durumları ifade eden bir kip kastediliyor). Var olan özgür kişiler üzerinden anlatılmak isteneni somutlayalım. Argümana göre özgür iradeleri gereği şu anda var olan tüm insanların şu andan itibaren yalnızca ahlaki kötülük yapmaları mümkündür. Bu oldukça olasılık dışı olabilir ancak argümanın iddiasına göre insanların liberteryen anlamıyla özgür olmaları bunu metafiziksel açıdan mümkün kılar. Fakat özgür iradeden Sapkınlık veya Sapkınlık-benzeri gibi böyle bir durumun imkanına bu geçiş hızlı bir sıçramadır. Tanrı varsa ahlaken yanlış ilk birkaç eylemlerinden sonra insanların özgür iradelerini kötü gidişata engel olmak adına ellerinden alabilir. Üstelik bu fiili olarak özgür irade sahibi olmadığımız anlamına da gelmez zira Tanrı bunu fiili olmayan ve muhtemelen asla olmayacak, tüm insanların daima ahlaken kötü olanı seçtikleri dünyalarda yapar.

Bir önceki argümanın ilk itirazına benzer olarak basit ilk cevap: tüm bunlar doğru olsa dahi (30), (31) için bir zemin sağlar; Tanrı varsa argüman zaten benzer ateistik argümanlar gibi yanlıştır ama bu kendi başına hangi imkan öncülünün doğru olduğu tartışmasının sonucunu belirlemede bir ağırlığı olmadığı anlamına gelmez.

İkincisi, bazı liberteryen özgür iradeli eylemler eş zamanlı gerçekleşiyor olabilir. Eş zamanlı özgür iradeli eylemler, her biri ahlaken kötü olması durumunda Sapkınlık-benzeri’nin örneklenmesine yetecek sayıdaysa Tanrı kötü gidişe özgürlüğe müdahale etmeden engel olamaz.

Üçüncü ve daha mühimi, argümanın diyalektik gücü itirazın ifade ettiğinden çok daha fazladır. Neden olduğunu görmek için itirazdaki örneğe yeniden bakalım. Argüman hatırına bir t anına dek gerçekleştirilmiş özgür seçimlerin ağırlıkla ahlaken iyi olduğunu kabul edelim. t’ye kadar dünya fiili kılınmaya değerdir. İtirazın iddiasına göre dünyanın buraya kadarki kısmından dallanan ve insanlar tarafından ahlaki iyi ile kötü arasındaki dengenin kötü lehine çevrileceği seçimler yapılan dünyalar, denge henüz ahlaki kötülük lehine olmadan Tanrı tarafından engellenir. Sormamız gereken soru: t anına nasıl gelinmiştir? Ahlaki iyiliğin ağır bastığı herhangi bir döneme gelinmesi için başlangıçta en az bir özgür kişinin yaratılması zorunludur. Ancak başlangıç durumunda ne ahlaki iyi ne de ahlaki kötü diğerine ağır basar, henüz ahlaken önemli hiçbir seçim yapılmamıştır. Argümanda gördüğümüz üzere, ilk seçiminde –veya ilk üç, beş, yüz seçiminde- ahlaken doğru olanı gerçekleştirecek bir kişi fiili kılınsa dahi ahlaken kötünün gerçekleştirildiği, dolayısıyla da kötünün ağır bastığı mümkün dünyalar olacaktır. Bunun olmaması, ilk seçimin özgürce yapılmadığı anlamına gelir ki bu durumda ahlaki kötülüğün ağır bastığı dünyanın mümkün olmamasıyla birlikte iyinin ağır bastığı dünya da mümkün olmaz. Öyleyse fiili kılınan dünyanın dengenin ahlaki iyilik lehine olduğu herhangi bir anda olması için dengenin ahlaki kötülük lehine olduğu bazı dünyalar halihazırda mümkün olmalıdır.

İlk durumun incelenmesine dayanan cevap yeniden eleştirilebilir. Birçoğumuz özgür iradenin içsel olarak değerli olduğunu düşünürüz. Öyleyse belki de ilk birkaç seçimin ahlaken yanlış olduğu mümkün dünyalar, iradenin özgürlüğündeki bu içsel değerin ağır basması dolayısıyla yine de yaratılmaya değerdir -her şeye rağmen toplamda denge iyi lehinedir.

Ne var ki bu tarz bir karşı cevap da ciddi problemleri beraberinde getirir. İlk olarak, özgür iradenin değeri ne tür bir değerdir? Ona sahip olan varlık için bir tür mükemmel yapıcı nitelik olma anlamında değerli olmadığını düşünmek için, hızlıca, iki gerekçemiz var: kendisinden daha mükemmeli mümkün olmayan Tanrı, ahlaki kötülük yapamaması anlamında bu tarz bir özgür iradeye sahip değildir ve en mükemmel yer olarak cennet, ahlaki kötülük yapabilen insanlar içermeme anlamında yine bu tarz bir özgür iradenin getireceği değerden yoksundur. Özgür iradenin değeri muhtemelen ona sahip olanı/onu içereni değerli kılmasından başka bir şeydedir. Çölde hiçbir seçim yapmaksızın duran özgür irade sahibi bir kişinin değeri de muhtemelen özgür iradeye sahip olmasına dayanmayacaktır. Aradığımız değer daha ziyade eylemin özgür oluşu ile ilgilidir: değerli olan özgür irade sahibi kişi değil, özgür iradeli eylemlerdir. Ancak bu noktada yine bir sorunla karşı karşıyayız. Bir soyguncunun özgürce banka soymaya karar vermesi, kararı özgür iradesinden kaynaklanması dolayısıyla bir miktar değerliyse de kararın ahlaken yanlışlığının bu değere ağır basacağı kuşkusuzdur. Dolayısıyla özgür iradeli eylemin değerinin ahlaki bir kötülüğe ağır basacağı kesin değildir. Dahası, ahlaken kötü bir eylemin özgür iradeli olması durumunda olmamasına göre daha da kötü olduğunu da düşünebiliriz. Soyguncunun kendisine isteği dışında verilmiş bir maddenin etkisinde söz konusu kötülüğe yol açması, bunu kasten yapmasından daha kabul edilebilir görünüyor.

İkinci olarak, seçimin özgür oluşunun toplama her halükarda pozitif katkı yaptığını kabul etsek dahi yaratılan kişilerin ilk seçimlerinin, ahlaken yanlış olmaları durumunda bu pozitif katkıya net biçimde ağır basacakları ölçüde ahlaken oldukça önemli olacağını düşünmek için de sebebimiz var. Yaratılan kişiler her şeyden önce Tanrı’ya itaat etme ve baş kaldırma arasında bir seçimle karşı karşıya olacaklardır. Bu seçim ise, Havva ve yasak elma hikayesinde olduğu gibi, sonucu itibariyle büyük öneme sahiptir. Öyle ki birçok teist dünyadaki sefaleti, acıyı, diğer birçok kötülüğü Tanrı’dan yüz çevirmemizle ilişkilendirir. Yaratıldığında Tanrı’ya karşı sevgi ve itaat içeren bir ilişki kurmayı seçmek veya reddetmekte özgür olan insanların, argüman gösteriyor ki, bunu reddettiği mümkün dünyalar vardır. İradenin ve Tanrı ile ilişkinin şeklinin değeri hakkındaki incelememizin de işaret ettiği üzere böyle bir dünyada kötülüğün ağır basacağı barizdir. Sonuç olarak toplamda kötülüğün ağır bastığı, Tanrı’nın fiili kılmayacağı mümkün dünyalar olmalıdır.

Bir diğer itiraz, argümanın molinist temelleri hedef alınarak getirilebilir. Biri kırmızı diğeri mavi iki ışık ve bir butondan oluşan, butona basıldığında bu iki ışıktan birinin yandığı ancak hangisinin yanacağının belirli olmadığı (epistemik anlamda değil, nedensel bir belirlenimin yokluğu söz konusu) basit bir sistem düşünelim. Diyelim ki bir t anında butona basılmamış olsun. Eğer butona basılmış olsaydı hangi renkteki ışığın yanacağına dair bir doğrudan söz edebilir miyiz? Molinizmin eleştirmenlerine göre butona basılması hangi ışığın yanacağını belirlemediğine göre eğer basılmış olsaydı hangi ışığın yanacağına dair bir doğru da olmamalıdır. Bölümün başındaki örnek hatırlanacak olursa benzer bir durum söz konusudur: Smith liberteryen anlamda özgür olduğuna göre (dolayısıyla da seçimi önceden belirlenmediğine göre) eğer Smith’e $20.000 teklif edilmiş olsaydı teklife ne cevap vereceğine dair de bir doğru olmamalıdır. Molinizme göre bu tarz, yaratılmış varlıkların karşıolgusal koşullu özgür seçimleriyle ilgili önermeler (counterfactual conditionals of creaturely freedom) doğruluk değerine sahiptir ve dolayısıyla Tanrı’nın bilgisi bunun gibi önermeleri de kapsar. Tanrı’nın, yaratacağı kişilerin hangi C durumlarında hangi eylemleri yapacağını bildiğini ve bundan dolayı belirli mümkün dünyaları fiili olmaktan alıkoymaya kadir olduğunu ifade eden argüman, bu yüzden molinizmi kabul eder.

O halde molinizme yöneltilen temel itiraz geçerliyse argüman hatalıdır, Tanrı Sapkınlık’ı fiili olmaktan alıkoyamayabilir. Öyleyse itiraz hakkında ne söyleyebiliriz? Yeniden ifade etmek gerekirse itiraz, eylemlerin liberteryen özgür iradeli olmaları dolayısıyla söz konusu karşıolgusal koşullu önermeleri doğru kılabilecek bir temel olamayacağını söyler. Bu noktada tartışmanın özgür iradeli eylemlerin karşıolgusal durumlarda muhtemelen ne yönde olacağı ile ilgili olmadığına dikkat çekmeliyiz. Smith’in, eğer kendisine $20,000 teklif edilmiş olsaydı muhtemelen teklifi reddedeceğini söyleyebilecek konumda olabiliriz. Ancak ihtiyacımız olan bu değildir. Aranan, Smith’in$20.000’lık teklife ne yanıt vereceğine dair kesin bir doğrudur.

Plantinga, yaratılmış varlıkların karşıolgusal koşullu özgür seçimleriyle ilgili en azından bazı önermelerin doğru olduğunu göstermek için ilginç bir argüman öne sürmüştür.14 Örnekte Smith’e $35,000 teklif edildiğini ve Smith’in bunu kabul ettiğini hatırlayalım. Bu örnek üzerinden ilerleyen argümanda ise Smith’in $20,000’ı da kabul edip etmeyeceğini tartışıyorduk. Belki de bir açıdan kabul edip etmeyeceğine dair bir doğru yokmuş gibi görünmesinin nedeni, Smith’in karakterini veya seçimi yaptığı koşulları bilmiyor olmamızdır. Plantinga bunun yerine bu kez de $36,000 teklif edildiğini düşünmemizi ister. Böyle bir durumda

  • (39) Eğer $36,000 teklif edilmiş olsaydı Smith rüşveti kabul edecekti.

doğru görünecektir. Öyleyse tartıştığımız karşıolgusal özgürlük önermelerinden en azından bazılarının doğru olduğunu düşünmek için bir sebebe sahip olduğumuzu söyleyebiliriz.

(39)’u hala kabul etmememiz gerektiği şeklinde karşılık verilebilir. Nihayetinde Smith hala liberteryen anlamda özgürdür ve dolayısıyla seçimi belirlenmemiştir. Bu durumda bize doğru gibi görünen önerme (39) yerine aslında

  • (39*)  Eğer $36,000 teklif edilmiş olsaydı Smith rüşveti muhtemelen kabul edecekti.

olabilir mi? Bu açık görünmüyor. Pruss’un örnek verdiği gibi devlet $35,000 üstündeki rüşvetlere daha yüksek ceza uyguluyorsa veya Smith’in yedi ile bölünebilen sayılara özel bir sempatisi varsa $36,000’lık rüşveti reddedebilir. Ancak bir an için Smith’in $35,000’ı $36,000’a tercih etmek için bu tarz ek nedenlere sahip olmadığını ve hala daha fazla paraya değer verdiğini farz edelim. (39)’u yine de karalayabilir miyiz? Öyle görünüyor ki özgür irade hakkında liberteryenizmin özgür eylemlerin rastgele olmadığını ifade edebilmesi için eylemlerin değerlendirmeler tarafından belirlenmiş (determined by) olmasalar da değerlendirmeler dolayısıyla (because of) gerçekleştiğini kabul etmesi gerekmektedir.15 Bundan dolayı, öyle görünüyor ki, liberteryen özgür iradenin söz konusu olması (39)’un olumsallığına işaret eder ancak makul bir liberteryen özgür irade anlayışı için benimsenecek pozisyon, (39)’un doğruluğunu ima eder.

Karşıolgusal özgürlük önermelerinin doğru olamayacağını öne süren itiraza farklı bir açıdan da yaklaşabiliriz. Tanrı’nın bilgisinin gelecekteki özgür eylemleri kapsamadığını düşünenler teizm içerisinde azınlıktadır. Yaygın görüşe göre Tanrı gelecekte gerçekleşecekleri, gelecekteki özgür iradeli eylemler de dahil olmak üzere bilir. Öyleyse yaratılmış varlıkların gelecekteki özgür eylemleri ile ilgili önermelere doğruluk değerlerini veren nedir? Bir diğer deyişle, aşağıdaki gibi bir önermeyi ne doğru veya yanlış kılmaktadır?

  • (40) Bill, yarın sabah kahvaltıda peynirli omlet yiyecektir.

Neden bunun yerine

  • (41) Bill, yarın sabah kahvaltıda sandviç yiyecektir.

doğru değildir? Argüman için doğru olan önermenin (40) olduğunu kabul edelim. Karşıolgusal önermelere dair itirazın sahip olduğu fikir hattı burada da söz konusudur. Bill peynirli omlet ile sandviç arasından seçim yapmakta liberteryen anlamda özgürdür ve dolayısıyla yapacağı seçim önceden belirlenmemiştir. Seçim yapacağı an da henüz fiili değildir. O halde (40) neden doğrudur? (41) yerine onu doğru kılanın, basitçe, Bill’in yarın sabah kahvaltıda peynirli omlet yiyecek olması olduğunu söyleriz. Aynısının karşıolgusal önermeler için de geçerli olduğunu düşünmemiz makuldür: (23)’ü doğru kılan da, basitçe, Smith’in kendisine $20,000 teklif edilseydi teklifi kabul edecek oluşudur. Gelecekle ilgili doğru önermeler doğrudur çünkü ifade ettikleri durum, zamanı geldiğinde vuku bulacaktır. Benzer olarak karşıolgusal koşullu önermeler doğrudur çünkü ifade ettikleri durum, koşullu önermenin karşıolgusal öncül kısmı doğru olsaydı gerçekleşecekti. Önemli bir nokta: (40) ve (41) gibi gelecekteki özgür eylemlere dair önermeler olumsal doğrular veya yanlışlardır. Bu, yeniden, özgür iradenin liberteryen olmasından ileri gelir. Yine benzer olarak (23) ve (24) gibi karşıolgusal özgürlük önermeleri de olumsal doğrular veya yanlışlar olacaktır. Ancak olumsallığının (40)’ı doğru olmaktan alıkoymaması gibi (23)’ün benzer sebeple niçin doğru olamayacağını düşünmemiz gerektiği de açık değildir.

Gelecekteki özgür iradeli eylemlerle ilgili önermelerin karşıolgusal özgürlük önermelerinden bir açıdan farklı olduğu, ilkinin –ikincisinin aksine- doğru olup olmadığının zamanı geldiğinde belli olacağı söylenebilse de bu sorunun özüyle ilgisizdir. Gelecekteki özgür iradeli eylemler biz belirlemeden önce, şu an doğrudur. Yine bu önermeler, onları doğru kılan durumlar henüz fiili değilken de doğrudur. Tüm bunlar karşıolgusal özgürlük önermelerine paraleldir. Sonuç olarak karşıolgusal özgürlük önermelerinin doğru olabileceğini kabul etmek, klasik teizmin gelecekteki özgür iradeli eylemleri Tanrı’nın bilgisine dahil eden ön bilgi geleneğini kabul etmek ölçüsünde makuldür.

4. Sonuç

Tanrı’nın varlığı üzerine devam eden tartışmada yeni bir alan açan ve teizmi, teizmin imkan dahilinde oluşu ve ateizmi de ateizmin imkan dahilinde oluşu üzerinden temellendirme olanağı tanıyan modal ontolojik argümanın (ve ters modal ontolojik argümanın) imkanla ilgili öncülü birçok felsefe disiplininden yararlanan argümanlarla tartışılmıştır. Burada detaylı olarak incelemesek de argümanın diğer öncülü de tartışılmaya değerdir. Örneğin modal ontolojik argüman klasik anlayışta S5 aksiyomuna dayandırılıyor olsa ve biz de bu anlayışı takip etmiş olsak da argümanın S5 kullanılmadan da formülize edilebileceğini düşünen felsefeciler vardır. Yapısı gereği iki muhtelif argüman arasında gerçekten ilkin bir simetri olup olmadığı da burada tartışmadığımız ancak değerlendirilmesi gereken bir konudur. Yalnızca diğer öncül ve argümanın yapısı değil, imkan öncülü hakkında ortaya koyduğumuz değerlendirmeler de elbette eksiksiz değildir. Hem teistik imkan öncülü lehine Gödel, Maydole, Buras ve Cantrell gibi hem de ateistik imkan öncülü lehine Guleserian, Bass, Stringer gibi isimlerin argümanları incelenmelidir. Tüm bu eksikliklerden dolayı nihai olarak olmasa da yazı kapsamında ateistik imkan öncülü lehine argümanların –teistik imkan öncülü lehine olanların aksine- iki ontolojik argüman arasındaki simetriyi ters modal ontolojik argüman lehine bozduğu sonucuna ulaşıyoruz.

Notlar

  • [1] Vaidya, Anand, “The Epistemology of Modality”, The Stanford Encyclopedia of Philosophy (Winter 2017 Edition), Edward N. Zalta (ed.), URL = <https://plato.stanford.edu/archives/win2017/entries/modality-epistemology/>. (Erişim tarihi 10.01.2020).
  • [2] Plantinga, Alvin, The Nature of Necessity, (Oxford: Oxford University Press, 1974), s. 45.
  • [3] Bkz. Nagasawa, Yujin, Maximal God: A New Defence of Perfect Being Theism, Oxford: OxfordUniversity Press, 2017), s. 187.
  • [4] Bkz. Chalmers, David J. (2002), “Does Conceivability Entail Possibility?”, Gendler, T. S. ve Hawthorne, J. (der.), Conceivability and Possibility, Oxford: Clarendon Press, ss. 145-200. Chalmers iki ayrım daha yapsa da ilki amacımız açısından yeterli olduğu için onunla yetiniyoruz.
  • [5] Pruss, Alexander (2001), “Śaṃkara’s Principle and Two Ontomystical Arguments”, International Journal for Philosophy of Religion 49, ss. 111-20.
  • [6] Kordig, Carl R. (1981), “A Deontic Argument for God’s Existence”, Nous 15, ss. 207-8.
  • [7] Johnson, Daniel M. (2009), “The Sense of Deity and Begging the Question with Ontological and Cosmological Arguments”, Faith and Philosophy 26:1, ss. 87-94.
  • [8] Pruss, Alexander (2010), “The Ontological Argument and the Motivational Centers of Lives”, Religious Studies 46, ss. 233-49.
  • [9] Maximal God: A New Defence of Perfect Being Theism, ss. 202-205.
  • [10] Rasmussen, Joshua (2018), “Plantinga”, Ontological Arguments, Graham Oppy (der.), Cambridge University Press, Cambridge, 2018, ss. 176–194.
  • [11] van Inwagen, Peter. (1988) “The Magnitude, Duration, and Distribution of Evil: A Theodicy,” Philosophical Topics 16, ss. 161-187.
  • [12] Plantinga, Alvin, God Freedom and Evil, (Grand Rapids, Michigan: William B. Eerdmans Publishing
  • Company, 1974), s. 30.
  • [13] Richard Gale, On the Nature and Existence of God (New York, NY: Cambridge University Press, 1996), ss. 224-237.
  • [14] Nature of Necessity, s. 177.
  • [15] Değerlendirmelerle kastedilenlere nelerin dahil edilebileceği ayrıca tartışılabilir. Karakter özellikleri, inançlar, eğilimler vb. örnek olabilir.

Not: Bu içerik ilk kez 2020 yılında yayınlanan Öncül Analitik Felsefe Dergisi’nin (tükenen) 4. sayısında yayınlanmıştır.

Öncül Analitik Felsefe Dergisi, 19 Ocak 2018 tarihinde kuruldu. Sunum, söyleşi, makale, çeviri, canlı yayın gibi içerikler üreterek Analitik Felsefe’ye dair Türkçe veritabanını genişletmeye devam ediyor.

Bir cevap yazın

Your email address will not be published.

Önceki Gönderi

Mümkün Dünyalara LPL (Leibniz, Plantinga, Lewis) Kamerasından Bakmak: Gottfried Leibniz – Zafer Ketizmen

En Güncel Haberler Analitik Felsefe:Tümü